От чего зависит величина индукционного тока: От чего зависит величина индукционного тока?

Содержание

От чего зависит величина индукционного тока: получение переменной ЭДС

Принцип получения переменной ЭДС

Пусть в однородном магнитном поле постоянного магнита равномерно вращается рамка (рис. 2.1), активные стороны которой перпендикулярны чертежу и пересекающие линии магнитной индукции с некоторой линейной скоростью v по часовой стрелке. При этом в сторонах а и в рамки наводится ЭДС противоположной полярности. При пересечении линии между точками А и В в сторонах рамки полярность меняется на противоположную.

Рис. 2.1

Время Т одного полного изменения ЭДС (это время одного оборота рамки) называют периодом ЭДС. Изменение ЭДС со временем может быть изображено на временной диаграмме (рис. 2.2). Магнитный поток Ф через рамку будет равен:

Ф = ВS cos б, 2. 1

где В — вектор магнитной индукции;

S — площадь активной части рамки;

сos б — угол между нормалью к рамке n и вектором магнитной индукции.

Для исчерпывающего определения синусоиды достаточно указать ее амплитуду, период и начальную фазу. Рекомендуем читателю самостоятельно построить две-три синусоиды с разными начальными фазами.

Рис. 2.2

Необходимость определения начальной фазы. Необходимость определения начальной фазы вытекает из следующего простого примера.

Представьте себе два последовательно включенных генератора, частоты и амплитуды ЭДС которых одинаковы. Спрашивается: можно ли заранее определить, суммарное напряжение в каждый момент времени? Очевидно, что нельзя.

Согласное включение двух генераторов. Если генераторы имеют одинаковую начальную фазу, то кривые напряжения (синусоиды в данном случае) для каждого генератора, изображенные на одном и том же чертеже и в одинаковом масштабе, совпадут. Следовательно, общее (суммарное) напряжение обоих генераторов будет всегда вдвое больше напряжения каждого генератора в отдельности. Обычно такое включение генераторов называют согласным.

Встречное включение двух генераторов. Предположим, что один генератор имеет начальную фазу, равную нулю, а другой — равную 180°, т. е. величина напряжения первого генератора в любой момент времени имеет то же значение, что и величина напряжения второго, однако знаки напряжений («+» или «-«) не будут совпадать.

В момент времени, когда напряжение первого генератора положительно, напряжение второго генератора отрицательно, и наоборот.

Учитывая, что напряжения складываются алгебраически, приходим к выводу, что результирующее напряжение в каждый момент времени равно нулю. Заметим, что если фазы генераторов отличаются на 180°, то иногда говорят, что генераторы работают в противофазе или соединены встречно.

Сдвиг фазы. Очевидно, что две синусоиды, имеющие разные начальные фазы, как бы сдвинуты одна относительно другой по горизонтали. Поэтому разность начальных фаз двух синусоид и называют обычно сдвигом фазы.

На рис. 2.3 показано простейшее устройство для получения переменного тока. По катушке проходит постоянный ток, и, следовательно, магнитное поле также постоянно. Стальной сердечник придает магнитным линиям желательную форму: между полюсами получается приблизительно однородное поле. В этом поле равномерно вращается прямоугольная рамка. Концы рамки соединены при помощи скользящих контактов с вольтметром.

Как уже сказано, магнитный поток, созданный катушкой, является постоянным. Но та его доля, которая сцеплена с вращающейся рамкой, будет неодинакова в разные моменты времени.

Изменение величины магнитного потока, пронизывающего виток, происходит непрерывно, хотя поток, создаваемый электромагнитом, остается неизменным. Следовательно, в рамке будет наводиться ЭДС. И действительно, опыт показывает, что стрелка вольтметра отклоняется.

Рис. 2.3 Получение переменного тока

Стрелка вольтметра попеременно отклоняется вправо и влево от нулевого положения.

Поскольку при вращении рамки пересекающий ее магнитный поток все время меняется, то по закону электромагнитной индукции в ней будет наводиться ЭДС индукции. Если период измеряется в секундах, то частота измеряется в герцах. В большинстве стран, включая Россию, промышленная частота переменного тока составляет 50 Гц (в США и Японии — 60 Гц). Величина промышленной частоты переменного тока обусловлена технико-экономическими соображениями. Если она слишком низка, то увеличиваются габариты электрических машин и, следовательно, расход материалов на их изготовление; заметным становится мигание света в электрических лампочках. При слишком высоких частотах увеличиваются потери энергии в сердечниках электрических машин и трансформаторах. Поэтому наиболее оптимальными оказались частоты 50 — 60 Гц. Однако в некоторых случаях используются переменные токи как с более высокой, так и с более низкой частотой. Например, в самолетах применяется частота 400 Гц. На этой частоте можно значительно уменьшить габариты и вес трансформаторов и электромоторов, что для авиации более существенно, чем увеличение потерь в сердечниках.

Катушка индуктивности является пассивным компонентом электронных схем, основное предназначение которой является сохранение энергии в виде магнитного поля. Свойство катушки индуктивности чем-то схоже с конденсатором, который хранит энергию в виде электрического поля.

Индуктивность (измеряется в Генри) — это эффект возникновения магнитного поля вокруг проводника с током. Ток, протекающий через катушку индуктивности, создает магнитное поле, которое имеет связь с электродвижущей силой (ЭДС) оказывающее противодействие приложенному напряжению.

Возникающая противодействующая сила (ЭДС) противостоит изменению переменного напряжения и силе тока в катушке индуктивности. Это свойство индуктивной катушки называется индуктивным сопротивлением. Следует отметить, что индуктивное сопротивление находится в противофазе к емкостному реактивному сопротивлению конденсатора в цепи переменного тока. Путем увеличения числа витков можно повысить индуктивность самой катушки.

Индуктивность в электрических цепях

В то время как конденсатор оказывает сопротивление изменению переменного напряжения, индуктивность же сопротивляется переменному тока. Идеальная индуктивность не будет оказывать сопротивление постоянному току, однако, в реальности все индуктивные катушки сами по себе обладают определенным сопротивлением.

В целом, отношение между изменяющимися во времени напряжением V(t) проходящим через катушку с индуктивностью L и изменяющимся во времени током I(t), проходящим через нее можно представить в виде дифференциального уравнения следующего вида:

Когда переменный синусоидальной ток (АС) протекает через катушку индуктивности, возникает синусоидальное переменное напряжение (ЭДС). Амплитуда ЭДС зависит от амплитуды тока и частоте синусоиды, которую можно выразить следующим уравнением:

где ω является угловой частотой резонансной частоты F:

Причем, фаза тока отстает от напряжения на 90 градусов. В конденсаторе же все наоборот, там ток опережает напряжение на 90 градусов. Когда индуктивная катушка соединена с конденсатором (последовательно либо параллельно), то образуется LC цепь, работающая на определенной резонансной частоте.

Индуктивное сопротивление ХL определяется по формуле:

где ХL — индуктивное сопротивление, ω — угловая частота, F — частота в герцах, и L индуктивность в генри.

Индуктивное сопротивление — это положительная составляющая импеданса. Оно измеряется в омах. Импеданс катушки индуктивности (индуктивное сопротивление) вычисляется по формуле:

Применение катушек индуктивности

Индуктивности широко используются в аналоговых схемах и схемах обработки сигналов. Они в сочетании с конденсаторами и другими радиокомпонентами образуют специальные схемы, которые могут усилить или отфильтровать сигналы определенной частоты.

Катушки индуктивности получили широкое применение начиная от больших катушек индуктивности, таких как дроссели в источниках питания, которые в сочетании с конденсаторами фильтра устраняют остаточные помехи и другие колебания на выходе источника питания, и до столь малых индуктивностей, которые располагаются внутри интегральных микросхем.

Две (или более) катушки индуктивности, которые соединены единым магнитным потоком, образуют трансформатор, являющимся основным компонентом схем работающих с электрической сетью электроснабжения. Эффективность трансформатора возрастает с увеличением частоты напряжения.

По этой причине, в самолетах используется переменное напряжение с частотой 400 герц вместо обычных 50 или 60 герц, что в свою очередь позволяет значительно сэкономить на массе используемых трансформаторов в электроснабжении самолета.

Так же индуктивности используются в качестве устройства для хранения энергии в импульсных стабилизаторах напряжения, в высоковольтных электрических системах передачи электроэнергии для преднамеренного снижения системного напряжения или ограничения ток короткого замыкания.

Законы Фарадея и Ленца

Электрические токи создают магнитные эффекты. А возможно ли, чтобы магнитное поле порождало электрическое? Фарадей обнаружил, что искомые эффекты возникают вследствие изменения МП во времени.

Когда проводник пересекается переменным магнитным потоком, в нем индуцируется электродвижущая сила, вызывающая электроток. Системой, которая генерирует ток, может быть постоянный магнит или электромагнит.

Явление электромагнитной индукции регулируется двумя законами: Фарадея и Ленца.

Закон Ленца позволяет охарактеризовать электродвижущую силу относительно ее направленности.

Важно! Направление индуцированной ЭДС такое, что вызванный ею ток стремится противостоять создающей его причине.

Фарадей заметил, что интенсивность индуцированного тока растет, когда быстрее изменяется число силовых линий, пересекающих контур. Другими словами, ЭДС электромагнитной индукции находится в прямой зависимости от скорости движущегося магнитного потока.

ЭДС индукции

Формула ЭДС индукции определена как:

Е = — dФ/dt.

Знак «-» показывает, как полярность индуцированной ЭДС связана со знаком потока и меняющейся скоростью.

Получена общая формулировка закона электромагнитной индукции, из которой можно вывести выражения для частных случаев.

Вращающаяся катушка

Работа генератора электроэнергии основана на вращении контура в МП, имеющего N витков.

ЭДС индуцируется в электроцепи всегда, когда магнитный поток ее пересекает, в соответствии с определением магнитного потока Ф = B x S х cos α (магнитная индукция, умноженная на поверхностную площадь, через которую проходит МП, и косинус угла, образованного вектором В и перпендикулярной линией к плоскости S).

Из формулы следует, что Ф подвержен изменениям в следующих случаях:

  • меняется интенсивность МП – вектор В;
  • варьируется площадь, ограниченная контуром;
  • изменяется ориентация между ними, заданная углом.

В первых опытах Фарадея индуцированные токи были получены путем изменения магнитного поля В. Однако можно индуцировать ЭДС, не двигая магнит или не меняя ток, а просто вращая катушку вокруг своей оси в МП. В данном случае магнитный поток меняется из-за изменения угла α. Катушка при вращении пересекает линии МП, возникает ЭДС.

Если катушка вращается равномерно, это периодическое изменение приводит к периодическому изменению магнитного потока. Или количество силовых линий МП, пересекаемых каждую секунду, принимает равные значения с равными интервалами времени.

Вращение контура в МП

Важно! Наведенная ЭДС меняется вместе с ориентацией с течением времени от положительной до отрицательной и наоборот. Графическое представление ЭДС представляет собой синусоидальную линию.

Для формулы ЭДС электромагнитной индукции применяется выражение:

Е = В х ω х S x N x sin ωt, где:

  • S – площадь, ограниченная одним витком или рамкой;
  • N – количество витков;
  • ω – угловая скорость, с которой вращается катушка;
  • В – индукция МП;
  • угол α = ωt.

На практике в генераторах переменного тока часто катушка остается неподвижной (статор), а электромагнит вращается вокруг нее (ротор).

Взаимоиндукция

Если две катушки расположены рядом, то в них наводится ЭДС взаимоиндукции, зависящая от геометрии обеих схем и их ориентации относительно друг друга. Когда разделение цепей возрастает, взаимоиндуктивность снижается, так как уменьшается соединяющий их магнитный поток.

Взаимоиндукция

Пусть имеется две катушки. По проводу одной катушки, обладающей N1 витками, протекает ток I1, создающий МП, проходящее через катушку с N2 витками. Тогда:

  1. Взаимоиндуктивность второй катушки относительно первой:

М21 = (N2 x F21)/I1;

  1. Магнитный поток:

Ф21 = (М21/N2) x I1;

  1. Найдем индуцированную ЭДС:

Е2 = — N2 x dФ21/dt = — M21x dI1/dt;

  1. Идентично в первой катушке индуцируется ЭДС:

Е1 = — M12 x dI2/dt;

Важно! Электродвижущая сила, вызванная взаимоиндукцией в одной катушке, всегда пропорциональна изменению электротока в другой.

Взаимную индуктивность можно признать равной:

М12 = М21 = М.

Соответственно, E1 = — M x dI2/dt и E2 = M x dI1/dt.

М = К √ (L1 x L2),

где К – коэффициент связи между двумя индуктивностями.

Явление взаимоиндукции используется в трансформаторах – электроаппаратах, позволяющих изменить значение напряжения переменного электротока. Аппарат представляет собой две катушки, намотанные вокруг одного сердечника. Ток, присутствующий в первой, создает меняющееся МП в магнитопроводе и электроток в другой катушке. Если количество витковых оборотов первой обмотки меньше, чем другой, напряжение увеличивается, и наоборот.

Кроме генерирования, трансформации электроэнергии магнитная индукция применяется в иных устройствах. Например, в магнитных левитационных поездах, которые двигаются не в непосредственном контакте с рельсами, а на несколько сантиметров выше из-за электромагнитной силы отталкивания.

>Видео

Если через замкнутый проводник или контур будет изменяться магнитный поток Ф, а значит магнитное поле, то в нем будет возникать ЭДС индукции и электрический ток.
Источниками переменного магнитного поля являются:
а) движущийся постоянный магнит;
б)движущийся проводник с постоянным током;
в) неподвижный проводник с переменным током.
Причем, чем быстрее изменяется магнитный поток магнитного поля через контур, тем большая ЭДС индукции, а значит и сила тока возникает в нем, таким образом, величина ЭДС индукции зависит от скорости изменения магнитного потока через контур, т.е.
– это и будет законом электромагнитной индукции, для замкнутого проводника или контура:
“ЭДС индукции ?инд, возникающая в контуре прямо пропорциональна скорости изменения магнитного потока через него”.
Электромагнитная индукция – это создание ЭДС индукции в проводнике, а если он замкнут, то и электрического тока в нем, под действием переменного магнитного поля.
Закон электромагнитной индукции для катушки:
“ЭДС индукции возникающая в катушке прямо пропорциональна числу витков катушки и скорости изменения магнитного потока через нее”. Если прямолинейный проводник будет двигаться в однородном магнитном поле равномерно, то в нем будет возникать ЭДС индукции, величина которой будет определяться по формуле:
где l – длина проводника, – скорость его движения, – угол между направлением и .

“ЭДС индукции возникающая в прямолинейном проводнике равномерно движущимся в однородном магнитном поле с индукцией , прямо пропорциональна длине проводника, скорости его движения, синусу угла между направлениями магнитной индукции магнитного поля и скорости движения проводника в магнитном поле ”.
Знак «-» в законе электромагнитной индукции указывает на направление индукционного тока в замкнутом проводнике, определяемое по правилу Ленца: “Индукционный ток всегда имеет такое направление, что своим магнитным полем препятствует изменению магнитного поля, которое вызвало индукционный ток”. Если магнитное поле, вызвавшее индукционный ток нарастает или увеличивается через замкнутый проводник, то магнитное поле индукционного тока направлено ему противоположно и не будет давать ему нарастать, если же магнитное поле, вызвавшее индукционный ток убывает, то магнитное поле индукционного тока направленно также, как и убывающее магнитное поле и препятствует убыванию магнитного поля вызывающего индукционный ток.
Величина индукционного поля определяется по закону Ома: “Сила индукционного тока прямо пропорциональна ЭДС индукции возникающей в замкнутом проводнике и обратно пропорциональна сопротивлению замкнутого проводника”.
Индукционным током называется ток, полученный в замкнутом проводнике под действием переменного магнитного поля, т.е. благодаря электромагнитной индукции.
Проводник с переменным током в пространстве вокруг себя образует переменное магнитное поле, в котором будет находиться сам проводник с переменным током, поэтому в нем будет возникать ЭДС индукции, которую называют ЭДС самоиндукции, а явление электромагнитной индукции в проводнике с переменным током называют самоиндукцией.
Таким образом, самоиндукция – это частный случай электромагнитной индукции, который имеет место в проводнике с переменным током. Величина ЭДС самоиндукции, возникающая в проводнике с переменным током определяется по закону самоиндукции: “ЭДС самоиндукции, возникающая в проводнике с переменным током прямо пропорциональна скорости изменения силы тока, текущего в проводнике”.
где – скорость изменения силы тока в проводнике, характеризует быстроту изменения тока в проводнике, и показывает на сколько изменяется ток за единицу времени;
L — индуктивность проводника – это характеристика проводника по которому течет переменный ток зависит от размеров и формы проводника, и не зависит от его материала.
L показывает какая ЭДС самоиндукции возникает в проводнике при скорости изменения тока в нем равной единице или какая ?сам возникает в проводнике, если ток в нем изменяется на единицу за единицу времени.
СИ: – это индуктивность такого проводника, в котором возникает ЭДС самоиндукции равная 1 В при изменении тока на 1 А за 1 секунду.
Знак «-» в законе самоиндукции указывает на направление тока самоиндукции, определяемого по правилу Ленца. Например, при замыкании цепи ток увеличивается, возникает ЭДС самоиндукции, которая препятствует нарастанию тока; если же ток уменьшается, что происходит при размыкании цепи, то ЭДС самоиндукции будет поддерживать убывающий ток, не давая ему убывать, поэтому при размыкании цепи большой индуктивности искрит рубильник из-за большого тока самоиндукции, определяемого по закону Ома для тока самоиндукции: .
Вариант №1
1. В каком случае в катушке замкнутой на гальванометр возникает электрический ток?
2. Что определяет закон электромагнитной индукции?
3. От чего зависит ЭДС индукции, возникающая в катушке?

4. В замкнутую катушку вдвигают постоянный магнит: один раз быстро, второй медленно. В каком случае в ней возникнет больший индукционный ток?
5. Что определяют по правилу Ленца? по правилу левой руки? правого винта?
6. Из формул
а)силы Ампера выразить силу тока;
б)силы Лоренца выразить скорость движения частиц;
в) закона индукции электромагнитной индукции выразить изменение магнитного потока.
7. Что такое самоиндукция? Где она имеет место?
8. В каких единицах измеряются
а)магнитная индукция;
б)магнитный поток;
в)ЭДС индукции;
г)индуктивность проводника.
Вариант №2
1. В чем заключается явление электромагнитной индукции?
2. Какое поле создает ЭДС индукции в замкнутом проводнике? Разве может магнитное поле заставить двигаться неподвижные электрические заряды?
3. В каком случае может возникать, а в каком не возникать, ЭДС индукции в прямолинейном проводнике, движущимся в однородном магнитном поле?
4. Почему в проводнике с постоянным током не возникает ЭДС индукции, а в проводнике с переменным возникает?
5. За 3 секунды магнитный поток, пронизывающий проволочную рамку, равномерно увеличивается с 6 Вб до 9 Вб. Чему равно при этом значение ЭДС индукции в рамке?
6. Чему равна ЭДС самоиндукции в катушке индуктивностью L = 3 Гн, при равномерном уменьшении силы тока от 5 А до 1 А за 2 секунды?
Вариант №3
1. Как направлена сила Лоренца, действующая на движущуюся заряженную частицу в магнитном поле?
2. Как направлена магнитная индукция внешнего магнитного поля, в котором против часовой стрелки движется отрицательно заряженная частица?
3. Электрон влетел в магнитное поле со скоростью V перпендикулярно линиям магнитной индукции и стал двигаться по окружности радиуса R. Определите величину вектора магнитной индукции магнитного поля. (заряд электрона e, m – масса электрона)
4. Под каким углом расположен проводник с током 3 А длиной 0,1 м в однородном магнитном поле с индукцией 4 Тл, если на него действует сила Ампера равная 0,6 Н?
5. Что происходит с индукционным током в кольце, плоскость которого перпендикулярна линиям магнитной индукции, если ее величина равномерно увеличивается?
6. Чему равна индуктивность проволочной рамки, если при силе тока I = 3 А в рамке возникает магнитный поток Ф = 6 Вб?
7. Катушка сопротивлением 100 Ом, состоящая из 1000 витков площадью 5 см2, внесена в однородное магнитное поле. В течение некоторого времени индукция магнитного поля уменьшилась от 0,8 Тл до 0,3 Тл. Какой заряд будет индуцирован в проводнике за это время?


явление электромагнитной индукции(опыты Фарадея)



явление электромагнитной индукции(опыты Фарадея)

32)Явление электромагнитной индукции.

В 1831 г. Фарадей открыл, что во всяком замкнутом проводящем контуре при изменении потока магнитной индукции через поверхность, ограниченную этим контуром, возникает электрический ток. Это явление называют электромагнитной индукцией, а возникающий ток индукционным.

Рис. 105.

Величина индукционного тока не зависит от способа, которым вызывается изменение потока магнитной индукции Ф, но определяется лишь скоростью изменения Ф, т. е. значением dФ/dt. При изменении знака dФ/dt меняется также направление тока. Поясним сказанное следующим примером. На рис. 105 изображен контур 1, силу тока в котором i1 можно менять с помощью реостата. Ток i1 создает магнитное поле, пронизывающее контур 2. Если увеличивать ток i1 поток магнитной индукции Ф через контур 2 будет расти. Это приведет к появлению в контуре 2 индукционного тока i2, регистрируемого гальванометром. Уменьшение тока i1 обусловит убывание потока магнитной индукции через второй контур, что приведет к появлению в нем индукционного тока иного направления, чем в первом случае. Индукционный ток i2 можно вызвать также, приближая контур 2 к первому контуру, или удаляя второй контур от первого. В обоих случаях направления возникающего тока будут противоположными. Наконец, электромагнитную индукцию можно вызвать, не перемещая контур 2 поступательно, а поворачивая его так, чтобы менялся угол между нормалью к контуру и направлением поля.

Заполнение всего пространства, в котором поле отлично от нуля, однородным магнетиком приводит, при прочих равных условиях, к увеличению индукционного тока в μ раз. Этим подтверждается то, что индукционный ток обусловлен изменением не потока вектора Н, а потока магнитной индукции.

Ленц установил правило, с помощью которого можно найти направление индукционного тока. Правило Ленца гласит, что индукционный ток всегда направлен так, чтобы противодействовать причине, его вызывающей. Если, например, изменение Ф вызвано перемещением контура, то возникает индукционный ток такого направления, что сила, действующая на него во внешнем поле, противится движению контура. При приближении контура 2 к первому контуру возникает ток i2 (рис. 105), магнитный момент которого направлен против внешнего поля (угол α между векторами рm и В равен π). Следовательно, согласно формуле (48.8) на контур 2 будет действовать сила, отталкивающая его от первого контура. При удалении контура 2 от первого контура возникает ток i2″, момент которого рm совпадает по направлению с В (α = 0), так что сила, действующая на контур 2, имеет направление к первому контуру.

Пусть контур 2 неподвижен, и ток индуцируется в нем путем изменения тока i1 в первом контуре. В этом случае индуцируется ток i2 такого направления, что создаваемый им собственный магнитный поток стремится ослабить изменения внешнего потока, приведшие к появлению индукционного тока. При увеличении i1 т. е. возрастании внешнего магнитного потока, направленного вправо, возникнет ток i2”, создающий поток, направленный влево. При уменьшении i1 возникает ток I, собственный магнитный поток которого направлен так же, как и внешний поток, и, следовательно, стремится поддержать внешний поток неизменным.

ЯВЛЕНИЕ ЭЛЕКТРО-МАГ. ИНДУКЦИИ. ПРАВИЛО ЛЕНЦА.

Явление электромагнитной индукции заключается в том, что в замкнутом проводящем контуре при изменении потока магнитной индукции, охватываемой этим контуром возникает электрический ток, называемый индукционным потоком. Эксперементально установленно, что величина индукции тока не зависит от способов изменения магнитного потока, а определяет лишь скорость изменения. Индукционный ток в проводнике может возникнуть только под действием ЭДС. ЭДС, возникшая в проводнике при изменении магнитного потока, называется ЭДС-индукции. Согласно закону Фарадея: ε инд=k dФm/dt. Направление индукции тока определяется по правилу Ленца. При всяком изменении магнитного потока сквозь поверхность натянутую на замкнутый контур, в нем возникает индукционный ток такого направления, что его магнитное поле препятствует изменению магнитного потока. С учетом правила Ленца закон Фарадея имеет вид: ε инд = — dФm/dt; Поскольку для замкнутого контура dФm=dψ, то ε инд = — dψ/dt


21.индукционный ток

6.2. Индукционный ток. Индукционный заряд.

Вихревое электрическое поле

Рассмотрим явление электромагнитной индукции, возникающее в короткозамкнутой катушке. Пусть катушка содержит N витков общим сопротивлением R. Если потокосцепление катушки изменяется во времени, то в катушке появляется ЭДС индукции:

.

Сила индукционного тока, возникающего в катушке, равна

.

За время существования в катушке индукционного тока (от момента времени до момента ) по катушке пройдет индуцированный (индукционный) электрический заряд

, (6.4)

где и – значения потокосцепления катушки в начальный и конечный моменты времени наблюдения. Важно, что величина индукционного заряда не зависит от времени наблюдения, а определяется лишь начальным и конечным потокосцеплениями.

Проиллюстрируем этот вывод следующим примером. Пусть потокосцепление катушки дважды меняется от значения до значения с разными постоянными скоростями , а время наблюдения электромагнитной индукции определяется интервалами () и (). Тогда получаем следующий график изменения потокосцепления (рис. 6.7). Зависимость силы индукционного тока от времени в двух случаях, согласно закону электромагнитной индукции, показана на рис. 6.8. Поскольку начальные и конечные значения потокосцеплений одинаковы, то, согласно (6.4), индукционный заряд, прошедший через катушку в двух случаях, один и тот же: . Поскольку величина прошедшего заряда определяется площадью под графиком зависимости силы тока от времени, то заштрихованные на рис. 6.8 площади одинаковы.

Если в проводящем контуре при изменении магнитного потока через него возникает ЭДС, то величина ЭДС определяется суммарной удельной работой сторонних сил по переносу зарядов в контуре. Поскольку электрические заряды могут перемещаться под действием сил со стороны электрического поля, то

. (6.5)

Если закон электромагнитной индукции (6.3) дает отличное от нуля значение , то из этого следует, что возникающее стороннее электрическое поле не потенциально (циркуляция его напряженности по замкнутому контуру отлична от нуля). Следовательно, это электрическое поле не является кулоновским. Каковы его принципиальные отличия от изученного нами ранее электростатического поля?

Во-первых, это электрическое поле создается не электрическими зарядами, а переменным во времени магнитным потоком.

Во-вторых, силовые линии такого поля не являются разомкнутыми. Они замкнуты сами на себя, т.е. рассматриваемое нами электрическое поле является вихревым. Поясним сказанное. Если переменное во времени магнитное поле попадает в сплошную проводящую среду (некоторый проводящий контур), то замкнутые силовые линии возникающего вихревого электрического поля определяют направление движения зарядов в среде, т.е. направление индукционного тока (рис.6.9). Однако возникает вопрос: будет ли наблюдаться явление электромагнитной индукции, если переменное магнитное поле не встречает проводящей среды? “Знает” ли магнитное поле о существовании свободных носителей заряда на своем пути? Очевидно, что в этом случае вихревое электрическое поле также возникает, и его можно зафиксировать, исследовав поведение в пространстве его силовых линий (рис.6.10). В частности, если поместить в пространство проводящий контур, совпадающий с силовыми линиями вихревого поля, то в нем появится индукционный ток.

Это можно пояснить еще одним примером. Рассмотрим взаимное сближение проводящего контура и постоянного магнита (рис.6.11). Если мы связываем рассмотрение с системой отсчета , то наблюдаем движение контура с некоторой скоростью вправо. Тогда свободные электроны кольца движутся в магнитном поле, и дальнейшее появление индукционного тока можно объяснить действием на них силы со стороны магнитного поля (см. п.6.1). Однако если рассмотреть процесс в системе отсчета , связанной с кольцом, то появление индукционного тока нельзя будет связать с действием на электроны магнитных сил, т.к. они неподвижны в этой системе отсчета. Тем не менее, индукционный ток существует в кольце независимо от способа объяснения его появления. Следовательно, поскольку движение электрически заряженных частиц может быть вызвано либо действием на них магнитных сил (которые в системе (xyz) отсутствуют), либо действием электрических сил, то следует предположить возникновение некоторого вихревого электрического поля. Именно его действие на заряды кольца и вызывают появление индукционного тока. Заметим, что такое объяснение годится для любой системы отсчета, следовательно, является универсальным.

Объединим выражения (6.3) и (6.5):

. (6.6)

Фигурирующий в основном законе электромагнитной индукции магнитный поток сквозь ограниченную контуром поверхность может изменяться по ряду причин – благодаря изменению формы контура и его расположения в поле, а также из-за переменности самого поля. Полная производная учитывает все эти причины. В случае неподвижного контура поток может изменяться только при изменении магнитной индукции во всех точках неподвижной поверхности S, натянутой на контур L, с течением времени. В таком случае закон (6.6) записывают в виде

. (6.7)

Исследование индукционного тока в двигающих транспортных средствах Текст научной статьи по специальности «Физика»

«При выполнении упражнений происходит активная мыслительная деятельность учащихся, что в свою очередь приводит к эффективному непроизвольному запоминанию определений, свойств и признаков изучаемых фигур. Определения, свойства и признаки рассматриваемых фигур периодически повторяются в процессе выполнения разнообразных упражнений, что приводит в итоге к продуктивному запоминанию. Большое значение имеет и то, что учащиеся с большим удовольствием предпочитают выполнять эти упражнения, чем отвечать на теоретические вопросы» [4, с. 1].

Проведение уроков геометрии с использованием задач по готовым чертежам применяются мною в школе в течение трех лет (8,9,10-й классы). Большинство ребят уверенно решают задачи по готовым чертежам, приводят обоснованные ответы и могут успешно применять их как элементы содержательных задач. Внешний мониторинг качества образования в 9-м классе (результаты основного государственного экзамена) показал, что все учащиеся (33 чел.) справились с геометрическими задачами базового уровня, из них 79% выполнили работу на «4» и «5», и 45 % ребят решили геометрические задания повышенного уровня. Выявлено, что для 63% ребят методика проведения уроков с использованием упражнений на готовых чертежах помогла в освоении программного материала по геометрии. Это позволило мне убедиться в эффективности данной методики.

Таким образом, методика проведения уроков с использованием упражнений на готовых чертежах развивает пространственное воображение, способствует активизации мыслительной деятельности учащихся, развивает речь, память, логику рассуждений, учит делать правильное выводы, а значит является эффективным средством усвоения и закрепления теоретического материала по геометрии. Список использованной литературы:

1. Э. Балаян Геометрия. Задачи на готовых чертежах для подготовки к ЕГЭ. 10-11 классы http: //www .labirint.ru/reviews/goods/366719/

2. Э. Балаян Геометрия. Задачи на готовых чертежах для подготовки к ГИА и ЕГЭ 7-9 классы/ Ростов-на-Дону: Феникс, 2012

3. Е.М. Рабинович Математика.Задачи и упражнения на готовых чертежах. 10-11 классы. Геометрия. -М.: Илекса, 2006, 80с.

4. Н.Ф. Гаврилова Универсальные поурочные разработки по геометрии: 9 класс. М.:ВАКО, 2010, 320с.

© Доброва Н.В., 2016

УДК-537.312.8

Б.А.Каландаров

магистрант 1 курса физико-технического факультета Ходжентского Госуниверситета имени академика Б.Гафурова

Г.О.Рахимова

магистрантка 1 курса физико-технического факультета Ходжентского Госуниверситета имени академика Б.Гафурова

Научный руководитель: М.Шерматов к.ф.м.н., профессор кафедры общей физики и физики твердого тела

ХГУ имени академика Б.Гафурова

ИССЛЕДОВАНИЕ ИНДУКЦИОННОГО ТОКА В ДВИГАЮЩИХ ТРАНСПОРТНЫХ СРЕДСТВАХ

Аннотация

Статьи Шерматова М., Каландарова Б.А., и Рахимовой Г.О. на тему «Исследование индукционного тока в двигающих транспортных средствах».

В статье обсуждаются результаты экспериментального исследования появления индукционного электрического тока в двигающих транспортных средствах, по длине которых протянуто проводник

_МЕЖДУНАРОДНЫЙ НАУЧНЫЙ ЖУРНАЛ «СИМВОЛ НАУКИ» №7/2016 ISSN 2410-700X_

различной длины.

Показано, что при движении транспорных средств, над которыми проведен проводник, в проводнике возникает индукционный ток различной величины, который зависит от длины проводника и скорости перемещения транспорта, по мере увеличения длины проведника и скорости перемещения транспорта индукционный ток растет.

Ключевые слова

Индукционный ток, скорость, длина, перемещение, проводник, транспорт, средства, зависимость движения,

компас, полюса, магнитное поле, величина самолёт, поезд.

Изучением индукционного электрического тока ученые занимаются давно, внедраясь различным электрическим генераторам и измеряющим средствам, применение этого явления стало доступным даже для учеников средных школ.

Однако, любое направление науки для применения не имеет предела, поскольку индукционный ток может возникать не только при движении какого нибудь проводника в определенном магнитном поле, подобный электрический ток может быть обнаружен в подвижной и неподвижной воде [3,4]; даже установлено, что обычную неподвижную воду можно использоват как источник индукционного электрического тока, при этом считается, что молекулы и атомы в воде при хаотическом движении взаимодействуя с мировым, в том числе Земним магнитным полем превращаются в электричесие заряды, двигающийся в одно направление создают индукционный ток определенной величины.

Необходимо отметить, что в любой питьевой воде имеются определенное количество атомов и молекул примеси, которые играют определенную роль в создании индукционного тока в воде.

Окружающий нас воздух тоже состоит из большого количества разнообразных атомов и молекул, двигаюших хаотично, однако их скорость и растояние между ними больше чем молекул и атоми воды.

С точки зрения нанотехнологии и нанофизики каждый атом рассматривается как один наномир, в котором может происходит различные физические явления [2]. Вокруг любого атома могут возникать электрические и магнитные поля, взаимодействующие с магнитным полем Земного шара, а также с магнитным полем космических тел.

Транспортные средства при движении на поверхности Земли также пересекают магнитное поле мирового масштаба, при этом возникают индукционные токи.

Для выяснения величины индукционного тока в двигающем транспорте были проведены специальные исследования, с легковой автомашиной к которой через диэлектрик был присоединен проводник, при движении автомашины вместе с проводником пересекалось магнитное поле на поверхности Земли и измерялос создавший индукционный ток в проводнике.

На рисунке 1 приведена электрическая схема измерения, индукционного тока в проводнике, прикрепленный к автомашине, где микроамперметром измерялась сила индукционного тока, возникшей при перемещении автомашины с проводником из меди.

Рисунок 1 — Электрическая схема соединения проводника на транспорте для измерения индукционного тока, возникшей при перемещении автомашины с проводником из меди. ¡¡А — микроамперметр, к — ключ

На рисунке 2 приведена способ прикрепления проводника к автомашине, где видно, что медный проводник над автомашиной находится в изолированной от автомашины через диэлектрик и соединён к микроамперметру.

ДИЭЛЕКТРИК

ДИЭЛЕКТРИК

Рисунок 2 — Внешный вид транспорта в котором измеряется индукционный ток.

При перемещение автомобиля проводник пересекает магнитное поле земли при этом в нем образуется индукционный ток определенной величины, зависящий от скорости перемещения автомобиля, соответственно проводника.

Следует отметить, что направление движения автомобиля определялось, при помощи компаса и всегда автомобиль двигался перпендикулярно направлению северного и южного магнитного полюса Земли.

В зависимости от скорости перемещения автомобиля соответственно проводника сила индукционного тока изменяется, причем увеличение скорости перемещения проводника способствовало росту силы индукционного тока.

На рисунке 3 приведена зависимость силы индукционного тока в проводнике от скорости перемещения проводника.

Из рисунке 3 видно что зависимость индукционного тока от скорости перемещения проводника имеет линейный характер, причем увеличение длины проводника при одной и той же скорости вызывает большему росту силы индукционного тока, что свидетельствует о закономерности образования индукционного тока, при увеличении длины провода пересечение силовых линий магнитного поля Земли будет больше, поэтому сила индукционного тока пропорционально длине провода растет.

ил

А

20-

10_

15

0

10 20 30 40 50 60 70 80 90 100

Рисунок 3 — Зависимость индукционного тока от скорости перемещения транспорта над которым прикреплен проводник определенной длины £ 1 — £ = 2,5м 2 — £ = 3м 3 — £ = 4,5м.

Если построить зависимость силы индукционного тока в проводнике разной длины при постоянной скорости 3(£) получим прямолинейную зависимость. Это говорит о том, что по мере увеличения длины транспортных средств сила индукционного тока может расти.

На рисунке 4 приведена зависимость 3(£), где видно, что она имеет также линейных характер, с коэффициентом к, £ — длина проволокон, 3 — сила тока. 3 = К£

/лкА

При нашем измерении к = 18-, это означает, что при увеличении длины проводника,

метр

двигающего совместно с транспортом на один метр сила индукционного тока увеличивается на 18 микроампер на проведенном нами участке.

Известно, что длина различных транспортных средств изменяется от нескольких до сотни метров, в частности длина поездов, в виде эталона достигается до сотни метров, следовательно при движении такого транспорта можно получить индукционные токи довольно большой величины. Если же самолёт, двигающий со сверхзвуковой скоростью принимать как проводник, тогда в таком самолёте при полёте над землёй формируется токи огромной величины.

Следует отметить, что при направлении движения с юга на север магнитного полюса и наоборот индукционный ток не возникал, потому что проводник практически не пересекал силовые линии магнитного поля Земного шара. Учитывая это явление можно создать спидометр для транспортных средств двигающих перпендикулярно линии Север-юг магнитного полюса Земного шара, где по величине индукционного тока можно судить о скорости передвижения транспортного средства.

От сюда следует, что индукционный ток в двигающих транспортных средствах возникает тогда, когда направление движения транспорта не двигается по направлению север-юг или наоборот-юг-север. В зависимости от количестве проводов, протянутых вдоль транспорта и их длины индукционный ток при движении транспорта увеличивается. На величину подобного тока влияет и скорость передвижения транспорта с проводами.

Рисунок 4 — Зависимость индукционного тока от длины проводника, расположенный на транспорте,

двигающего с постоянной скоростью 100 км/час.

_МЕЖДУНАРОДНЫЙ НАУЧНЫЙ ЖУРНАЛ «СИМВОЛ НАУКИ» №7/2016 ISSN 2410-700X_

Необходимо отметить о том, что в зависимости от длины транспортного средства и скорости его передвижения полученный индукционный ток можно использовать для различных целей, в частности, полученный в результате перемещения проводника перпендикулярно линии север-юг можно использовать для подзарядки акуммилятора транспорта, а также по величине индукционного тока, возбужденного проводником определенной длины можно судить о величине скорости передвижения транспорта, в других случаях растянутый вдоль транспорта можно определить скорости изменения магнитных бур на поверхности Земного шара и т.п.

Кроме того, отклонение величины индукционного тока при перемещении транспортного средства, соответственно и проволоки проведенный вдол транспорта от нормальных величин тока может свидетельствовать об изменениях магнитного поля за транспортным средством, т.е. о состоянии окружающий транспорта воздуха и влияние магнитного поля на психическое состояние водителей и посажиров в нем.

Таким образом, из приведенных выше данных можно придти к следующим основным выводам:

1. В любом транспортном средстве земного, надземного, морского назначения можно получить индукционный ток определенной величины.

2. Величина индукционного тока, полученный при передвижении транспортных средств зависит от длины перетянутого по длине транспорта и скорости его перемешения.

3.й, нахустпатент TJ №611 15.03.2013.

4. Телеснин Р.В., Яковел В.Ф., Курс физики, электричество. Москва, «Просвещение», 1984г., 486стр.

© Каландаров Б.А., Рахимова Г.О., 2016

Индукционный ток — Какое направление индукционного тока? — Росиндуктор

ИНДУКЦИОННЫЙ ТОК — это электрический ток, возникающий при изменении потока магнитной индукции в замкнутом проводящем контуре. Это явление носит название электромагнитной индукции. Хотите узнать какое направление индукционного тока? Росиндуктор — это торговый информационный портал, где вы найдете информацию про ток.

Содержание

Индукционный ток правило

Определяющее направление индукционного тока правило звучит следующим образом: «Индукционный ток направлен так, чтобы своим магнитным полем противодействовать изменению магнитного потока, которым он вызван». Правая рука развернута ладонью навстречу магнит¬ным силовым линиям, при этом большой палец направлен в сторону движения проводника, а четыре пальца по-казывают, в каком направлении будет течь индукционный ток. Перемещая проводник, мы перемещаем вместе с проводчиком все электроны, заключенные в нем, а при перемещении в магнитном поле электрических зарядов на них будет действовать сила по правилу левой руки.

Направление индукционного тока

Направление индукционного тока, как и его величина, определяется правилом Ленца, в котором говорится, что направление индукционного тока всегда ослабляет действие фактора, возбудившего ток. При изменении потока магнитного поля через контур направление индукционного тока будет таким, чтобы скомпенсировать эти изменения. Когда магнитное поле возбуждающее ток в контуре создается в другом контуре, направление индукционного тока зависит от характера изменений: при увеличении внешнего тока индукционный ток имеет противоположное направление, при уменьшении — направлен в ту же сторону и стремиться усилить поток.

Индукционный ток в катушке

Катушка с индукционным током имеет два полюса (северный и южный), которые определяются в зависимости от направления тока: индукционные линии выходят из северного полюса. Приближение магнита к катушке вызывает появление тока с направлением, отталкивающим магнит. При удалении магнита ток в катушке имеет направление, способствующее притягиванию магнита.

Индукционный ток возникает

Индукционный ток возникает в замкнутом контуре, находящемся в переменном магнитном поле. Контур может быть как неподвижным (помещенным в изменяющийся поток магнитной индукции), так и движущимся (движение контура вызывает изменение магнитного потока). Возникновение индукционного тока обуславливает вихревое электрическое поле, которое возбуждается под воздействием магнитного поля.

Как создать индукционный ток

О том, как создать кратковременный индукционный ток можно узнать из школьного курса физики.

Для этого есть несколько способов:

  • — перемещение постоянного магнита или электромагнита относительно катушки,
  • — перемещение сердечника относительно вставленного в катушку электромагнита,
  • — замыкание и размыкание цепи,
  • — регулирование тока в цепи.

Сила индукционного тока

Основной закон электродинамики (закон Фарадея) гласит, что сила индукционного тока для любого контура равна скорости изменения магнитного потока, проходящего через контур, взятой со знаком минус. Сила индукционного тока носит название электродвижущей силы.

От чего зависит сила индукционного тока. Электромагнитная индукция

Учитель физики ГБОУ СОШ №58 г. Севастополя Сафроненко Н.И.

Тема урока: Опыты Фарадея. Электромагнитная индукция.

Лабораторная работа «Исследование явления электромагнитной индукции»

Цели урока : Знать/понимать: определение явления электромагнитной индукции. Уметь описывать и объяснять электромагнитную индукцию, уметь проводить наблюдения природных явлений, использовать простые измерительные приборы для изучения физических явлений.

— развивающая: развивать логическое мышление, познавательный интерес, наблюдательность.

— воспитательная: Формировать убеждённость в возможности познания природы, необходимость разумного использования достижений науки для дальнейшего развития человеческого общества, уважения к творцам науки и техники .

Оборудование : Электромагнитная индукция: катушка с гальванометром, магнит, катушка с сердечником, источник тока, реостат, катушка с сердечником по которой течет переменный ток, сплошное и кольцо с прорезью, катушка с лампочкой. Фильм о М.Фарадее.

Тип урока: комбинированный урок

Метод урока: частично-поисковый, объяснительно-иллюстративный

Домашнее задание:

§21(стр.90-93), устно отвечать на вопросы стр.90, тест 11 стр.108

Лабораторная работа

Исследование явления электромагнитной индукции

Цель работы : выяснить

1)при каких условиях в замкнутом контуре (катушке) возникает индукционный ток;

2)от чего зависит направление индукционного тока;

3)от чего зависит сила индукционного тока.

Оборудование : миллиамперметр, катушка, магнит

Ход урока.

Соедините концы катушки с клеммами миллиамперметра.

1. Выясните, что электрический ток (индукционный) в катушке возникает при изменении магнитного поля внутри катушки. Изменения магнитного поля внутри катушки можно вызвать, вдвигая магнит в катушку или удаляя его из неё.

А)Введите магнит южным полюсом в катушку, а затем удалите.

Б) Введите магнит северным полюсом в катушку, а затем удалите.

При движении магнита появился ток (индукционный) в катушке? (При изменении магнитного поля внутри катушки появился индукционный ток?)

2. Выясните, что направление индукционного тока зависит от направления движения магнита относительно катушки (вносят магнит или удаляют) и от того каким полюсом вносят или удаляют магнит.

А)Введите магнит южным полюсом в катушку, а затем удалите. Пронаблюдайте, что происходит со стрелкой миллиамперметра в обоих случаях.

Б) Введите магнит северным полюсом в катушку, а затем удалите. Пронаблюдайте, что происходит со стрелкой миллиамперметра в обоих случаях. Нарисуйте направления отклонения стрелки миллиамперметра:

Полюса магнита

В катушку

Из катушки

Южный полюс

Северный полюс

3. Выясните, что сила индукционного тока зависит от скорости движения магнита (скорости изменения магнитного поля в катушке).

Медленно вводите магнит в катушку. Пронаблюдайте за показаниями миллиамперметра.

Быстро вводите магнит в катушку. Пронаблюдайте за показаниями миллиамперметра.

Вывод.

Ход урока

Дорога к знаниям? Её легко понять. Ответить можно просто: «Вы ошибаетесь и ошибаетесь опять, но меньше, меньше с каждым разом. Я выражаю надежду, что сегодняшний урок будет ещё одним меньше на этой дороге знаний. Наш урок посвящён явлению электромагнитной индукции, которое открыл английский физик Майкл Фарадей 29 августа 1831 года. Редкий случай, когда дата нового замечательного открытия известна так точно!

Явление электромагнитной индукции – явление возникновения электрического тока в замкнутом проводнике (катушке) при изменении внешнего магнитного поля внутри катушки. Ток называется индукционным. Индукция — наведение, получение.

Цель урока: изучить явление электромагнитной индукции, т.е. при каких условиях в замкнутом контуре (катушке) возникает индукционный ток, выяснить от чего зависит направление и величина индукционного тока.

Одновременно с изучением материала будете выполнять лабораторную работу.

В начале 19 века (1820г.) после опытов датского учёного Эрстеда стало ясно, что электрический ток создаёт вокруг себя магнитное поле. Вспомним ещё раз этот опыт. (Ученик рассказывает опыт Эрстеда ). После этого встал вопрос о том, нельзя ли получить ток с помощью магнитного поля, т.е. произвести обратные действия. В первой половине 19 века учёные обратились именно к таким опытам: стали искать возможность создания электрического тока за счёт магнитного поля. М.Фарадей в своём дневнике записал: «Превратить магнетизм в электричество». И шёл к своей цели почти десять лет. Справился с задачей блестяще. Как напоминание о том, над чем ему всё время следует думать, он носил в кармане магнит. Этим уроком мы отдадим дань уважения великому учёному.

Вспомним Майкла Фарадея. Кто же он такой? (Ученик рассказывает о М.Фарадее ).

Сын кузнеца, разносчик газет, переплётчик книг, самоучка, самостоятельно изучивший физику и химию по книгам, лаборант выдающегося химика Деви и наконец учёный, проделал большую работу, проявил изобретательность, настойчивость, упорство пока не получил электрический ток с помощью магнитного поля.

Совершим путешествие в те далёкие времена и воспроизведём опыты Фарадея. Фарадея считают крупнейшим в истории физики экспериментатором.

N S

1) 2)

S N

Магнит вводили в катушку. При движении магнита в катушке регистрировался ток (индукционный). Первая схема была довольно простой. Во-первых, М.Фарадей использовал в опытах катушку с большим числом витков. Катушка была присоединена к прибору миллиамперметру. Нужно сказать, что в те далёкие времена не было достаточно хороших инструментов для измерения электрического тока. Поэтому пользовались необычным техническим решением: брали магнитную стрелку, располагали рядом с ней проводник, по которому протекал ток, и по отклонению магнитной стрелки судили о протекающем токе. Мы будем судить о токе по показаниям миллиамперметра.

Учащиеся воспроизводят опыт, выполняют п.1 в лабораторной работе. Обратили внимание, что стрелка миллиамперметра отклоняется от своего нулевого значения, т.е. показывает, что в цепи появился ток тогда, когда магнит движется. Стоит магниту остановиться, как стрелка возвращается в нулевое положение, т.е.электрического тока в цепи нет. Ток появляется тогда, когда изменяется магнитное поле внутри катушки.

Пришли к тому о чём говорили в начале урока: получили электрический ток с помощью изменяющегося магнитного поля. Это первая заслуга М. Фарадея.

Вторая заслуга М. Фарадея — установил от чего зависит направление индукционного тока. Установим и мы это. Учащиеся выполняют п.2 в лабораторной работе. Обратимся к п.3 лабораторной работы. Выясним, что сила индукционного тока зависит от скорости движения магнита (скорости изменения магнитного поля в катушке).

Какие выводы сделал М.Фарадей?

    Электрический ток появляется в замкнутой цепи тогда, когда магнитное поле изменяется (если магнитное поле существует, но не меняется, то тока нет).

    Направление индукционного тока зависит от направления движения магнита и его полюсов.

    Сила индукционного тока пропорциональна скорости изменения магнитного поля.

Второй эксперимент М.Фарадея:

Взял две катушки на общем сердечнике. Одну подсоединил к миллиамперметру, а вторую с помощью ключа к источнику тока. Как только цепь замыкалась миллиамперметр показывал индукционный ток. Размыкалась тоже показывал ток. Пока цепь замкнута, т.е. в цепи идёт ток, миллиамперметр не показывал ток. Магнитное поле существует, но не меняется.

Рассмотрим современный вариант опытов М.Фарадея. В катушку соединённую с гальванометром вносим и выносим электромагнит, сердечник, включаем и выключаем ток, с помощью реостата меняем силу тока. На сердечник катушки, по которой течёт переменный ток надевают катушку с лампочкой.

Выяснили условия возникновения в замкнутой цепи (катушке) индукционного тока. А что является причиной его возникновения? Вспомним условия существования электрического тока. Это: заряженные частицы и электрическое поле. Дело в том, что изменяющееся магнитное поле порождает в пространстве электрическое поле (вихревое), которое действует на свободные электроны в катушке и приводит их в направленное движение, создавая таким образом индукционный ток.

Изменяется магнитное поле, изменяется количество силовых линий магнитного поля через замкнутый контур. Если вращать рамку в магнитном поле, то в ней появится индукционный ток. Показать модель генератора.

Открытие явления электромагнитной индукции имело огромное значение для развития техники, для создания генераторов, с помощью которых вырабатывается электрическая энергия, которые стоят на энергетических промышленных предприятиях (электростанциях). Демонстрируется фильм о М.Фарадее «От электричества до электрогенераторов» с 12.02 минуты.

На явлении электромагнитной индукции работают трансформаторы, с помощью которых передают электроэнергию без потерь. Демонстрируется линия электропередачи.

Явление электромагнитной индукции используется в работе дефектоскопа, с помощью которого исследуют стальные балки, рельсы (неоднородности в балке искажают магнитное поле и в катушке дефектоскопа возникает индукционный ток).

Хочется вспомнить слова Гельмгольца: «Пока люди будут пользоваться благами электричества, они будут помнить имя Фарадея».

«Да будут святы те, кто в творческом пылу, исследуя весь мир, открыли в нём законы».

Я думаю, что на нашей дороге знаний ошибок стало ещё меньше.

Что нового узнали? (Что ток можно получить с помощью изменяющегося магнитного поля. Выяснили от чего зависит направление и величина индукционного тока).

Чему научились? (Получать индукционный ток с помощью изменяющегося магнитного поля).

Вопросы:

    В металлическое кольцо в течении первых двух секунд вдвигают магнит, в течении следующих двух секунд он неподвижен внутри кольца, в течении следующих двух секунд его вынимают. В каких промежутках времени в катушке идёт ток? (От 1-2с; 5-6с).

    На магнит надевают кольцо с прорезью и без. В каком возникает индукционный ток? (В замкнутом кольце)

    На сердечнике катушки, которая подключена к источнику переменного тока, находится кольцо. Включают ток и кольцо подпрыгивает. Почему?

Оформление доски:

«Превратить магнетизм в электричество»

М.Фарадей

Портрет М.Фарадея

Рисунки опытов М.Фарадея.

Электромагнитная индукция – явление возникновения электрического тока в замкнутом проводнике (катушке) при изменении внешнего магнитного поля внутри катушки.

Этот ток называется индукционным.

Если в магнитном поле находится замкнутый проводящий контур, не содержащий источников тока, то при изменении магнитного поля в контуре возникает электрический ток. Это явление называется электромагнитной индукцией. Появление тока свидетельствует о возникновении в контуре электрического поля, которое может обеспечить замкнутое движение электрических зарядов или, другими словами, о возникновении ЭДС. Электрическое поле, которое возникает при изменении поля магнитного и работа которого при перемещении зарядов по замкнутому контуру не равна нулю, имеет замкнутые силовые линии и называется вихревым.

Для количественного описания электромагнитной индукции вводится понятие магнитного потока (или потока вектора магнитной индукции) через замкнутый контур. Для плоского контура, расположенного в однородном магнитном поле (а только такие ситуации и могут встретиться школьникам на едином государственном экзамене), магнитный поток определяется как

где — индукция поля, — площадь контура, — угол между вектором индукции и нормалью (перпендикуляром) к плоскости контура (см. рисунок; перпендикуляр к плоскости контура показан пунктиром). Единицей магнитного потока в международной системе единиц измерений СИ является Вебер (Вб), который определяется как магнитный поток через контур площади 1 м 2 однородного магнитного поля с индукцией 1 Тл, перпендикулярной плоскости контура.

Величина ЭДС индукции , возникающая в контуре при изменении магнитного потока через этот контур, равна скорости изменения магнитного потока

Здесь — изменение магнитного потока через контур за малый интервал времени . Важным свойством закона электромагнитной индукции (23.2) является его универсальность по отношению к причинам изменения магнитного потока: магнитный поток через контур может меняться из-за изменения индукции магнитного поля, изменения площади контура или изменения угла между вектором индукции и нормалью, что происходит при вращении контура в поле. Во всех этих случаях по закону (23.2) в контуре будет возникать ЭДС индукции и индукционный ток.

Знак минус в формуле (23.2) «отвечает» за направление тока, возникающего в результате электромагнитной индукции (правило Ленца). Однако понять на языке закона (23.2), к какому направлению индукционного тока приведет этот знак при том или ином изменении магнитного потока через контур, не так-то просто. Но достаточно легко запомнить результат: индукционный ток будет направлен таким образом, что созданное им магнитное поле будет «стремиться» компенсировать то изменение внешнего магнитного поля, которое этот ток и породило. Например, при увеличении потока внешнего магнитного поля через контур в нем возникнет индукционный ток, магнитное поле которого будет направлено противоположно внешнему магнитному полю так, чтобы уменьшить внешнее поле и сохранить, таким образом, первоначальную величину магнитного поля. При уменьшении потока поля через контур поле индукционного тока будет направлено так же, как и внешнее магнитное поле.

Если в контуре с током ток в силу каких-то причин изменяется, то изменяется и магнитный поток через контур того магнитного поля, которое создано самим этим током. Тогда по закону (23.2) в контуре должна возникать ЭДС индукции. Явление возникновения ЭДС индукции в некоторой электрической цепи в результате изменения тока в самой этой цепи называется самоиндукцией. Для нахождения ЭДС самоиндукции в некоторой электрической цепи необходимо вычислить поток магнитного поля, создаваемого этой цепью через нее саму. Такое вычисление представляет собой сложную проблему из-за неоднородности магнитного поля. Однако одно свойство этого потока является очевидным. Поскольку магнитное поле, создаваемого током в цепи, пропорционально величине тока, то и магнитный поток собственного поля через цепь пропорционален току в этой цепи

где — сила тока в цепи, — коэффициент пропорциональности, который характеризует «геометрию» цепи, но не зависит от тока в ней и называется индуктивностью этой цепи. Единицей индуктивности в международной системе единиц СИ является Генри (Гн). 1 Гн определяется как индуктивность такого контура, поток индукции собственного магнитного поля через который равен 1 Вб при силе тока в нем 1 А. С учетом определения индуктивности (23.3) из закона электромагнитной индукции (23.2) получаем для ЭДС самоиндукции

Благодаря явлению самоиндукции ток в любой электрической цепи обладает определенной «инерционностью» и, следовательно, энергией. Действительно, для создания тока в контуре необходимо совершить работу по преодолению ЭДС самоиндукции. Энергия контура с током и равна этой работе. Необходимо запомнить формулу для энергии контура с током

где — индуктивность контура, — сила тока в нем.

Явление электромагнитной индукции широко применяется в технике. На нем основано создание электрического тока в электрических генераторах и электростанциях. Благодаря закону электромагнитной индукции происходит преобразование механических колебаний в электрические в микрофонах. На основе закона электромагнитной индукции работает, в частности, электрическая цепь, которая называется колебательным контуром (см. следующую главу), и которая является основой любой радиопередающей или радиопринимающей техники.

Рассмотрим теперь задачи.

Из перечисленных в задаче 23.1.1 явлений только одно есть следствие закона электромагнитной индукции — появление тока в кольце при проведении сквозь него постоянного магнита (ответ 3 ). Все остальное — результат магнитного взаимодействия токов.

Как указывалось во введении к настоящей главе, явление электромагнитной индукции лежит в основе работы генератора переменного тока (задача 23.1.2 ), т.е. прибора, создающего переменный ток, заданной частоты (ответ 2 ).

Индукция магнитного поля, создаваемого постоянным магнитом, уменьшается с увеличением расстояния до него. Поэтому при приближении магнита к кольцу (задача 23.1.3 ) поток индукции магнитного поля магнита через кольцо изменяется, и в кольце возникает индукционный ток. Очевидно, это будет происходить при приближении магнита к кольцу и северным, и южным полюсом. А вот направление индукционного тока в этих случаях будет различным. Это связано с тем, что при приближении магнита к кольцу разными полюсами, поле в плоскости кольца в одном случае будет направлено противоположно полю в другом. Поэтому для компенсации этих изменений внешнего поля магнитное поле индукционного тока должно быть в этих случаях направлено по-разному. Поэтому и направления индукционных токов в кольце будут противоположными (ответ 4 ).

Для возникновения ЭДС индукции в кольце необходимо, чтобы менялся магнитный поток через кольцо. А поскольку магнитная индукция поля магнита зависит от расстояния до него, то в рассматриваемом в задаче 23.1.4 случае поток через кольцо будет меняться, в кольце возникнет индукционный ток (ответ 1 ).

При вращении рамки 1 (задача 23.1.5 ) угол между линиями магнитной индукции (а, значит, и вектором индукции) и плоскостью рамки в любой момент времени равен нулю. Следовательно, магнитный поток через рамку 1 не изменяется (см. формулу (23.1)), и индукционный ток в ней не возникает. В рамке 2 индукционный ток возникнет: в положении показанном на рисунке, магнитный поток через нее равен нулю, когда рамка повернется на четверть оборота — будет равен , где — индукция, — площадь рамки. Еще через четверть оборота поток снова будет равен нулю и т.д. Поэтому поток магнитной индукции через рамку 2 изменяется в процессе ее вращения, следовательно, в ней возникает индукционный ток (ответ 2 ).

В задаче 23.1.6 индукционный ток возникает только в случае 2 (ответ 2 ). Действительно, в случае 1 рамка при движении остается на одном и том же расстоянии от проводника, и, следовательно, магнитное поле, созданное этим проводником в плоскости рамки, не изменяется. При удалении рамки от проводника магнитная индукция поля проводника в области рамки изменяется, меняется магнитный поток через рамку, и возникает индукционный ток

В законе электромагнитной индукции утверждается, что индукционный ток в кольце будет течь в такие моменты времени, когда изменяется магнитный поток через это кольцо. Поэтому пока магнит покоится около кольца (задача 23.1.7 ) индукционный ток в кольце течь не будет. Поэтому правильный ответ в этой задаче — 2 .

Согласно закону электромагнитной индукции (23.2) ЭДС индукции в рамке определяется скоростью изменения магнитного потока через нее. А поскольку по условию задачи 23.1.8 индукция магнитного поля в области рамки изменяется равномерно, скорость ее изменения постоянна, величина ЭДС индукции не изменяется в процессе проведения опыта (ответ 3 ).

В задаче 23.1.9 ЭДС индукции, возникающая в рамке во втором случае, вчетверо больше ЭДС индукции, возникающей в первом (ответ 4 ). Это связано с четырехкратным увеличением площади рамки и, соответственно, магнитного потока через нее во втором случае.

В задаче 23.1.10 во втором случае в два раза увеличивается скорость изменения магнитного потока (индукция поля меняется на ту же величину, но за вдвое меньшее время). Поэтому ЭДС электромагнитной индукции, возникающая в рамке во втором случае, в два раза больше, чем в первом (ответ 1 ).

При увеличении тока в замкнутом проводнике в два раза (задача 23.2.1 ), величина индукции магнитного поля возрастет в каждой точке пространства в два раза, не изменившись по направлению. Поэтому ровно в два раза изменится магнитный поток через любую малую площадку и, соответственно, и весь проводник (ответ 1 ). А вот отношение магнитного потока через проводник к току в этом проводнике, которое и представляет собой индуктивность проводника , при этом не изменится (задача 23.2.2 — ответ 3 ).

Используя формулу (23.3) находим в задаче 32.2.3 Гн (ответ 4 ).

Связь между единицами измерений магнитного потока, магнитной индукции и индуктивности (задача 23.2.4 ) следует из определения индуктивности (23.3): единица магнитного потока (Вб) равна произведению единицы тока (А) на единицу индуктивности (Гн) — ответ 3 .

Согласно формуле (23.5) при двукратном увеличении индуктивности катушки и двукратном уменьшении тока в ней (задача 23.2.5 ) энергия магнитного поля катушки уменьшится в 2 раза (ответ 2 ).

Когда рамка вращается в однородном магнитном поле, магнитный поток через рамку меняется из-за изменения угла между перпендикуляром к плоскости рамки и вектором индукции магнитного поля. А поскольку и в первом и втором случае в задаче 23.2.6 этот угол меняется по одному и тому же закону (по условию частота вращения рамок одинакова), то ЭДС индукции меняются по одному и тому же закону, и, следовательно, отношение амплитудных значений ЭДС индукции в рамках равно единице (ответ 2 ).

Магнитное поле, создаваемое проводником с током в области рамки (задача 23.2.7 ), направлено «от нас» (см. решение задач главы 22). Величина индукции поля провода в области рамки при ее удалении от провода будет уменьшаться. Поэтому индукционный ток в рамке должен создать магнитное поле, направленное внутри рамки «от нас». Используя теперь правило буравчика для нахождения направления магнитной индукции, заключаем, что индукционный ток в рамке будет направлен по часовой стрелке (ответ 1 ).

При увеличении тока в проводе будет возрастать созданное им магнитное поле и в рамке возникнет индукционный ток (задача 23.2.8 ). В результате возникнет взаимодействие индукционного тока в рамке и тока в проводнике. Чтобы найти направление этого взаимодействия (притяжение или отталкивание) можно найти направление индукционного тока, а затем по формуле Ампера силу взаимодействия рамки с проводом. Но можно поступить и по-другому, используя правило Ленца. Все индукционные явления должны иметь такое направление, чтобы компенсировать вызывающую их причину. А поскольку причина — увеличение тока в рамке, сила взаимодействия индукционного тока и провода должна стремиться уменьшить магнитный поток поля провода через рамку. А поскольку магнитная индукция поля провода убывает с увеличением расстояния до него, то эта сила будет отталкивать рамку от провода (ответ 2 ). Если бы ток в проводе убывал, то рамка притягивалась бы к проводу.

Задача 23.2.9 также связана с направлением индукционных явлений и правилом Ленца. При приближении магнита к проводящему кольцу в нем возникнет индукционный ток, причем направление его будет таким, чтобы компенсировать вызывающую его причину. А поскольку эта причина — приближение магнита, кольцо будет отталкиваться от него (ответ 2 ). Если магнит отодвигать от кольца, то по тем же причинам возникло бы притяжение кольца к магниту.

Задача 23.2.10 — единственная вычислительная задача в этой главе. Для нахождения ЭДС индукции нужно найти изменение магнитного потока через контур . Это можно сделать так. Пусть в некоторый момент времени перемычка находилась в положении, показанном на рисунке, и пусть прошел малый интервал времени . За этот интервал времени перемычка переместится на величину . Это приведет к увеличению площади контура на величину . Поэтому изменение магнитного потока через контур будет равно , а величина ЭДС индукции (ответ 4 ).

9.5. Индукционный ток

9.5.1. Тепловое действие индукционного тока

Возникновение ЭДС приводит к появлению в проводящем контуре индукционного тока , сила которого определяется по формуле

I i = | ℰ i | R ,

где ℰ i — ЭДС индукции, возникающая в контуре; R — сопротивление контура.

При протекании индукционного тока в контуре выделяется теплота , количество которой определяется одним из выражений:

Q i = I i 2 R t , Q i = ℰ i 2 t R , Q i = I i | ℰ i | t ,

где I i — сила индукционного тока в контуре; R — сопротивление контура; t — время; ℰ i — ЭДС индукции, возникающая в контуре.

Мощность индукционного тока вычисляется по одной из формул:

P i = I i 2 R , P i = ℰ i 2 R , P i = I i | ℰ i | ,

где I i — сила индукционного тока в контуре; R — сопротивление контура; ℰ i — ЭДС индукции, возникающая в контуре.

При протекании индукционного тока в проводящем контуре через площадь поперечного сечения проводника переносится заряд , величина которого вычисляется по формуле

q i = I i ∆t ,

где I i — сила индукционного тока в контуре; Δt — интервал времени, в течение которого по контуру течет индукционный ток.

Пример 21. Кольцо, изготовленное из проволоки с удельным сопротивлением 50,0 ⋅ 10 −10 Ом ⋅ м, находится в однородном магнитном поле с индукцией 250 мТл. Длина проволоки равна 1,57 м, а площадь ее поперечного сечения составляет 0,100 мм 2 . Какой максимальный заряд пройдет по кольцу при выключении поля?

Решение . Появление ЭДС индукции в кольце вызвано изменением потока вектора индукции, пронизывающего плоскость кольца, при выключении магнитного поля.

Поток индукции магнитного поля через площадь кольца определяется формулами:

  • до выключения магнитного поля

Ф 1 = B 1 S  cos α,

где B 1 — первоначальное значение модуля индукции магнитного поля, B 1 = 250 мТл; S — площадь кольца; α — угол между направлениями вектора магнитной индукции и вектора нормали (перпендикуляра) к плоскости кольца;

  • после выключения магнитного поля

Ф 2 = B 2 S  cos α = 0,

где B 2 — значение модуля индукции после выключения магнитного поля, B 2 = 0.

∆Ф = Ф 2 − Ф 1 = −Ф 1 ,

или, с учетом явного вида Ф 1 ,

∆Ф = −B 1 S  cos α.

Среднее значение ЭДС индукции, возникающей в кольце при выключении поля,

| ℰ i | = | Δ Ф Δ t | = | − B 1 S cos α Δ t | = B 1 S | cos α | Δ t ,

где ∆t — интервал времени, за который происходит выключение поля.

Наличие ЭДС индукции приводит к появлению индукционного тока; сила индукционного тока определяется законом Ома:

I i = | ℰ i | R = B 1 S | cos α | R Δ t ,

где R — сопротивление кольца.

При протекании индукционного тока по кольцу переносится индукционный заряд

q i = I i Δ t = B 1 S | cos α | R .

Максимальному значению заряда соответствует максимальное значение функции косинус (cos α = 1):

q i max = I i Δ t = B 1 S R .

Полученная формула определяет максимальное значение заряда, который пройдет по кольцу при выключении поля.

Однако для расчета заряда необходимо получить выражения, которые позволят найти площадь кольца и его сопротивление.

Площадь кольца — площадь круга радиусом r , периметр которого определяется формулой длины окружности и совпадает с длиной проволоки, из которой изготовлено кольцо:

l = 2πr ,

где l — длина проволоки, l = 1,57 м.

Отсюда следует, что радиус кольца определяется отношением

r = l 2 π ,

а его площадь —

S = π r 2 = π l 2 4 π 2 = l 2 4 π .

Сопротивление кольца задается формулой

R = ρ l S 0 ,

где ρ — удельное сопротивление материала проволоки, ρ = 50,0 × × 10 −10 Ом ⋅ м; S 0 — площадь поперечного сечения проволоки, S 0 = = 0,100 мм 2 .

Подставим полученные выражения для площади кольца и его сопротивления в формулу, определяющую искомый заряд:

q i max = B 1 l 2 S 0 4 π ρ l = B 1 l S 0 4 π ρ .

Вычислим:

q i max = 250 ⋅ 10 − 3 ⋅ 1,57 ⋅ 0,100 ⋅ 10 − 6 4 ⋅ 3,14 ⋅ 50,0 ⋅ 10 − 10 = 0,625 Кл = 625 мКл.

При выключении поля по кольцу проходит заряд, равный 625 мКл.

Пример 22. Контур площадью 2,0 м 2 и сопротивлением 15 мОм находится в однородном магнитном поле, индукция которого возрастает на 0,30 мТл в секунду. Найти максимально возможную мощность индукционного тока в контуре.

Решение . Появление ЭДС индукции в контуре вызвано изменением потока вектора индукции, пронизывающего плоскость контура, при изменении индукции магнитного поля с течением времени.

Изменение потока вектора индукции магнитного поля определяется разностью

∆Ф = ∆BS  cos α,

где ∆B — изменение модуля индукции магнитного поля за выбранный интервал времени; S — площадь, ограниченная контуром, S = 2,0 м 2 ; α — угол между направлениями вектора магнитной индукции и вектора нормали (перпендикуляра) к плоскости контура.

Среднее значение ЭДС индукции, возникающей в контуре, при изменении индукции магнитного поля:

| ℰ i | = | Δ Ф Δ t | = | Δ B S cos α Δ t | = Δ B S | cos α | Δ t ,

где ∆B /∆t — скорость изменения модуля вектора индукции магнитного поля с течением времени, ∆B /∆t = 0,30 мТл/с.

Появление ЭДС индукции приводит к появлению индукционного тока; сила индукционного тока определяется законом Ома:

I i = | ℰ i | R = Δ B S | cos α | R Δ t ,

где R — сопротивление контура.

Мощность индукционного тока

P i = I i 2 R = (Δ B Δ t) 2 S 2 R cos 2 α R 2 = (Δ B Δ t) 2 S 2 cos 2 α R .

Максимальному значению мощности индукционного тока соответствует максимальное значение функции косинус (cos α = 1):

P i max = (Δ B Δ t) 2 S 2 R .

Вычислим:

P i max = (0,30 ⋅ 10 − 3) 2 (2,0) 2 15 ⋅ 10 − 3 = 24 ⋅ 10 − 6 Вт = 24 мкВт.

Максимальная мощность индукционного тока в данном контуре равна 24 мкВт.

Возникновение в проводнике ЭДС индукции

Если поместить в проводник и перемещать его так, чтобы он при своем движении пересекал силовые линии поля, то в проводнике возникнет , называемая ЭДС индукции .

ЭДС индукции возникнет в проводнике и в том случае, если сам проводник останется неподвижным, а перемещаться будет магнитное поле, пересекая проводник своими силовыми линиями.

Если проводник, в котором наводится ЭДС индукции, замкнуть на какую-либо внешнюю цепь, то под действием этой ЭДС по цепи потечет ток, называемый индукционным током.

Явление индуктирования ЭДС в проводнике при пересечении его силовыми линиями магнитного поля называется электромагнитной индукцией .

Электромагнитная индукция — это обратный процесс, т. е. превращение механической энергии в электрическую.

Явление электромагнитной индукции нашло широчайшее применение в . На использовании его основано устройство различных электрических машин.

Величина и направление ЭДС индукции

Рассмотрим теперь, каковы будут величина и направление индуктированной в проводнике ЭДС.

Величина ЭДС индукции зависит от количества силовых линий поля, пересекающих проводник в единицу времени, т. е. от скорости движения проводника в поле.

Величина индуктированной ЭДС находится в прямой зависимости от скорости движения проводника в магнитном поле.

Величина индуктированной ЭДС зависит также и от длины той части проводника, которая пересекается силовыми линиями поля. Чем большая часть проводника пересекается силовыми линиями поля, тем большая ЭДС индуктируется в проводнике. И, наконец, чем сильнее магнитное поле, т. е. чем больше его индукция, тем большая ЭДС возникает в проводнике, пересекающем это поле.

Итак, величина ЭДС индукции, возникающей в проводнике при его движении в магнитном поле, прямо пропорциональна индукции магнитного поля, длине проводника и скорости его перемещения.

Зависимость эта выражается формулой Е = Blv,

где Е — ЭДС индукции; В — магнитная индукция; I — длина проводника; v — скорость движения проводника.

Следует твердо помнить, что в проводнике, перемещающемся в магнитном поле, ЭДС индукции возникает только в том случае, если этот проводник пересекается магнитными силовыми линиями поля. Если же проводник перемещается вдоль силовых линий поля, т. е. не пересекает, а как бы скользит по ним, то никакой ЭДС в нем не индуктируется. Поэтому приведенная выше формула справедлива только в том случае, когда проводник перемещается перпендикулярно магнитным силовым линиям поля.

Направление индуктированной ЭДС (а также и тока в проводнике) зависит от того, в какую сторону движется проводник. Для определения направления индуктированной ЭДС существует правило правой руки.

Если держать ладонь правой руки так, чтобы в нее входили магнитные силовые линии поля, а отогнутый большой палец указывал бы направление движения проводника, то вытянутые четыре пальца укажут направление действия индуктированной ЭДС и направление тока в проводнике.

Правило правой руки

ЭДС индукции в катушке

Мы уже говорили, что для создания в проводнике ЭДС индукции необходимо перемещать в магнитном поле или сам проводник, или магнитное поле. В том и другом случае проводник должен пересекаться магнитными силовыми линиями поля, иначе ЭДС индуктироваться не будет. Индуктированную ЭДС, а следовательно, и индукционный ток можно получить не только в прямолинейном проводнике, но и в проводнике, свитом в катушку.

При движении внутри постоянного магнита в ней индуктируется ЭДС за счет того, что магнитный поток магнита пересекает витки катушки, т. е. точно так же, как это было при движении прямолинейного проводника в поле магнита.

Если магнит опускать в катушку медленно, то возникающая в ней ЭДС будет настолько мала, что стрелка прибора может даже не отклониться. Если же, наоборот, магнит быстро ввести в катушку, то отклонение стрелки будет большим. Значит, величина индуктируемой ЭДС, а следовательно, и сила тока в катушке зависят от скорости движения магнита, т. е. от того, насколько быстро силовые линии поля пересекают витки катушки. Если теперь поочередно вводить в катушку с одинаковой скоростью сначала сильный магнит, а затем слабый, то можно заметить, что при сильном магните стрелка прибора будет отклоняться на больший угол. Значит, величина индуктируемой ЭДС, а следовательно, и сила тока в катушке зависят от величины магнитного потока магнита.

И, наконец, если вводить с одинаковой скоростью один и тот же магнит сначала в катушку с большим числом витков, а затем со значительно меньшим, то в первом случае стрелка прибора отклонится на больший угол, чем во втором. Значит, величина индуктируемой ЭДС, а следовательно, и сила тока в катушке зависят от числа ее витков. Те же результаты можно получить, если вместо постоянного магнита применять электромагнит.

Направление ЭДС индукции в катушке зависит от направления перемещения магнита. О том, как определять направление ЭДС индукции, говорит закон, установленный Э. X. Ленцем.

Закон Ленца для электромагнитной индукции

Всякое изменение магнитного потока внутри катушки сопровождается возникновением в ней ЭДС индукции, причем чем быстрее изменяется магнитный поток, пронизывающий катушку, тем большая ЭДС в ней индуктируется.

Если катушка, в которой создана ЭДС индукции, замкнута на внешнюю цепь, то по виткам ее идет индукционный ток, создающий вокруг проводника магнитное поле, в силу чего катушка превращается в соленоид. Получается таким образом, что изменяющееся внешнее магнитное поле вызывает в катушке индукционный ток, которой, в свою очередь, создает вокруг катушки свое магнитное поле — поле тока.

Изучая это явление, Э. X. Ленц установил закон, определяющий направление индукционного тока в катушке, а следовательно, и направление ЭДС индукции. ЭДС индукции, возникающая в катушке при изменении в ней магнитного потока, создает в катушке ток такого направления, при котором магнитный поток катушки, созданный этим током, препятствует изменению постороннего магнитного потока.

Закон Ленца справедлив для всех случаев индуктирования тока в проводниках, независимо от формы проводников и от того, каким способом достигается изменение внешнего магнитного поля.


При движении постоянного магнита относительно проволочной катушки, присоединенной к клеммам гальванометра, или при движении катушки относительно магнита возникает индукционный ток.

Индукционные токи в массивных проводниках

Изменяющийся магнитный поток способен индуктировать ЭДС не только в витках катушки, но и в массивных металлических проводниках. Пронизывая толщу массивного проводника, магнитный поток индуктирует в нем ЭДС, создающую индукционные токи. Эти так называемые распространяются по массивному проводнику и накоротко замыкаются в нем.

Сердечники трансформаторов, магнитопроводы различных электрических машин и аппаратов представляют собой как раз те массивные проводники, которые нагреваются возникающими в них индукционными токами. Явление это нежелательно, поэтому для уменьшения величины индукционных токов части электрических машин и сердечники трансформаторов делают не массивными, а состоящими из тонких листов, изолированных один от другого бумагой или слоем изоляционного лака.Свитый в катушку проводник замыкается на гальванометре (рис. 3.19). Если вдвигать в катушку постоянный магнит, то гальванометр покажет наличие тока в течение всего промежутка времени, пока магнит перемещается относительно катушки. При выдергивании магнита из катушки гальванометр показывает наличие тока противоположного направления. Изменения направления тока происходит при изменении вдвигаемого или выдвигаемого полюса магнита.

Аналогичные результаты наблюдались при замене постоянного магнита электромагнитом (катушкой с током). Если обе катушки закрепить неподвижно, но в одной из них менять значение тока, то в этот момент в другой катушке наблюдается индукционный ток.

ЯВЛЕНИЕ ЭЛЕКТРОМАГНИТНОЙ ИНДУКЦИИ состоит в возникновении электродвижущей силы (э.д.с.) индукции в проводящем контуре, через который меняется поток вектора магнитной индукции. Если контур является замкнутым, то в нем возникает индукционный ток.

Открытие явления электромагнитной индукции:

1) показало взаимосвязь между электрическим и магнитным полем ;

2) предложило способ получения электрического тока с помощью магнитного поля.

Основные свойства индукционного тока :

1. Индукционный ток возникает всегда, когда происходит изменение сцепленного с контуром потока магнитной индукции.

2. Сила индукционного тока не зависит от способа изменения потока магнитной индукции, а определяется лишь скоростью его изменения.

Опытами Фарадея было установлено, что величина электродвижущей силы индукции пропорциональна скорости изменения магнитного потока, пронизывающего контур проводника (закон электромагнитной индукции Фарадея)

Или , (3.46)

где (dF) – изменение потока в течении времени (dt).МАГНИТНЫМ ПОТОКОМ или ПОТОКОМ МАГНИТНОЙ ИНДУКЦИИ называется величина, которая определяется на основе следующего соотношения: (магнитный поток через поверхность площадью S ): Ф=ВScosα, (3.45), угол a – угол между нормалью к рассматриваемой поверхности и направлением вектора индукции магнитного поля

единица магнитного потока в системе СИ носит название вебер – [Вб=Тл×м 2 ].

Знак «–» в формуле означает, что э.д.с. индукции вызывает индукционный ток, магнитное поле которого противодействует всякому изменению магнитного потока, т.е. при >0 э.д.с. индукции e И

э.д.с. индукции измеряется в вольтах

Для нахождения направления индукционного тока существует правило Ленца (правило установлено в 1833 г.): индукционный ток имеет такое направление, что создаваемое им магнитное поле стремится компенсировать изменение магнитного потока, вызвавшее этот индукционный ток.

Например, если вдвигать северный полюс магнита в катушку, т. е. увеличивать магнитный поток через его витки, в катушке возникает индукционный ток такого направления, что на ближайшем к магниту конце катушки возникает северный полюс (рис.3.20). Итак, магнитное поле индукционного тока стремится нейтрализовать вызвавшее его изменение магнитного потока.

Не только переменное магнитное поле порождает индукционный ток в замкнутом проводнике, но и при движении замкнутого проводника длиной l в постоянном магнитном поле (В) со скоростью v в проводнике возникает эдс:

a (B Ùv) (3.47)

Как вы уже знаете, электродвижущая сила в цепи– это результат действия сторонних сил. При движении проводника в магнитном поле роль сторонних сил выполняет сила Лоренца (которая действует со стороны магнитного поля на движущийся электрический заряд). Под действием этой силы происходит разделение зарядов и на концах проводника возникает разность потенциалов. Э.д.с. индукции в проводнике является работой по перемещению единичных зарядов вдоль проводника.

Направление индукционного тока можно определитьпо правилу правой руки: Вектор В входит в ладонь, отведенный большой палец совпадает с направлением скорости проводника, а 4 пальца укажут направление индукционного тока.

Таким образом переменное магнитное поле вызывает появление индуцированного электрического поля. Оно не потенциально (в отличие от электростатического), т.к. работа по перемещению единичного положительного заряда равна э.д.с. индукции , а не нулю.

Такие поля называются вихревыми. Силовые линии вихревого электрического поля – замкнуты сами на себя, в отличие от линий напряженности электростатического поля.

Э.д.с. индукции возникает не только в соседних проводниках, но и в самом проводнике при изменении магнитного поля тока, идущего по проводнику. Возникновение э.д.с. в каком-либо проводнике при изменении в нем самом силы тока (следовательно, магнитного потока в проводнике) называется самоиндукцией, а ток, индуцируемый в этом проводнике, – током самоиндукции.

Ток в замкнутом контуре создает в окружающем пространстве магнитное поле, напряженность которого пропорциональна силе тока I. Поэтому магнитный поток Ф, пронизывающий контур, пропорционален силе тока в контуре

Ф=L×I, (3.48).

L – коэффициент пропорциональности, который носит название коэффициента самоиндукции, или, просто, индуктивности. Индуктивность зависит от размеров и формы контура, а также от магнитной проницаемости среды, окружающей контур.

В этом смысле индуктивность контура — аналог электрической емкости уединенного проводника, которая также зависит только от формы проводника, его размеров и диэлектрической проницаемости среды.

Единица индуктивности — генри (Гн) : 1Гн — индуктивность такого контура, магнитный поток самоиндукции которого при токе в 1А равен 1Вб (1Гн=1Вб/А=1В·с/А).

Если L=const, то э.д.с. самоиндукции можно представить в следующем виде:

, или , (3.49)

где DI (dI) – изменение тока в цепи, содержащей катушку индуктивности (или контур) L, за время Dt (dt). Знак «–» в этом выражении означает, что э.д.с. самоиндукции препятствует изменению тока (т. е. если ток в замкнутом контуре уменьшается, то э.д.с. самоиндукции приводит к возникновению тока того же направления и наоборот).

Одним из проявлений электромагнитной индукции является возникновение замкнутых индукционных токов в сплошных проводящих средах: металлических телах, растворах электролитов, биологических органах и т.д. Такие токи носят название вихревых токов или токов Фуко. Эти токи возникают при перемещении проводящего тела в магнитном поле и/или при изменении со временем индукции поля, в которое помещены тела. Сила токов Фуко зависит от электрического сопротивления тел, а также от скорости изменения магнитного поля.

Токи Фуко также подчиняются правилу Ленца : их магнитное поле направлено так, чтобы противодействовать изменению магнитного потока, индуцирующему вихревые токи.

Поэтому массивные проводники тормозятся в магнитном поле. В электрических машинах, для того чтобы минимизировать влияние токов Фуко, сердечники трансформаторов и магнитные цепи электрических машин собирают из тонких пластин, изолированных друг от друга специальным лаком или окалиной.

Вихревые токи вызывают сильное нагревание проводников. Джоулево тепло, выделяемое токами Фуко , используется в индукционных металлургических печах для плавки металлов, согласно закону Джоуля-Ленца .

10.3 Закон электромагнитной индукции Фарадея | Электромагнетизм

10.3 Закон электромагнитной индукции Фарадея (ESBPY)

Ток, индуцированный изменяющимся магнитным полем (ESBPZ)

В то время как удивительное открытие электромагнетизма Эрстедом проложило путь для более практического применения электричества, именно Майкл Фарадей дал нам ключ к практическому производству электричества: электромагнитная индукция .

Фарадей обнаружил, что когда он перемещал магнит рядом с проводом, на нем генерировалось напряжение.Если магнит удерживался в неподвижном состоянии, напряжение не генерировалось, оно существовало только во время движения магнита. Мы называем это напряжение индуцированной ЭДС (\ (\ mathcal {E} \)).

Контурная петля, подключенная к чувствительному амперметру, будет регистрировать ток, если он настроен, как показано на этом рисунке, и магнит перемещается вверх и вниз:

Магнитный поток

Прежде чем мы перейдем к определению закона электромагнитной индукции Фарадея и примерам, нам сначала нужно потратить некоторое время на изучение магнитного потока.Для петли площадью \ (A \) в присутствии однородного магнитного поля \ (\ vec {B} \) магнитный поток (\ (φ \)) определяется как: \ [\ phi = BA \ cos \ theta \] Где: \ begin {align *} \ theta & = \ text {угол между магнитным полем B и нормалью к петле в области A} \\ A & = \ text {область петли} \\ B & = \ text {магнитное поле} \ end {align *}

Единицей измерения магнитного потока является Вебер (Вб).

Вы можете спросить себя, почему включен угол \ (\ theta \). Поток зависит от магнитного поля, проходящего через поверхность. Мы знаем, что поле, параллельное поверхности, не может вызвать ток, потому что оно не проходит через поверхность. Если магнитное поле не перпендикулярно поверхности, то есть компонент, который перпендикулярен, и компонент, который параллелен поверхности. Параллельная составляющая не может вносить вклад в поток, только вертикальная составляющая может.

На этой диаграмме мы показываем, что магнитное поле под углом, отличным от перпендикулярного, может быть разбито на составляющие.Компонент, перпендикулярный поверхности, имеет величину \ (B \ cos (\ theta) \), где \ (\ theta \) — угол между нормалью и магнитным полем.

Закон электромагнитной индукции Фарадея

ЭДС \ (\ mathcal {E} \), создаваемая вокруг контура проводника, пропорциональна скорости изменения магнитного потока φ через площадь A контура. Математически это можно выразить как:

\ [\ mathcal {E} = -N \ frac {\ Delta \ phi} {\ Delta t} \]

где \ (\ phi = B · A \), а B — напряженность магнитного поля.\ (N \) — количество контуров схемы. Магнитное поле измеряется в теслах (Тл). Знак минус указывает направление и то, что наведенная ЭДС имеет тенденцию противодействовать изменению магнитного потока. Знак минус можно игнорировать при вычислении звездных величин.

Закон Фарадея связывает наведенную ЭДС со скоростью изменения магнитного потока, который является произведением магнитного поля и площади поперечного сечения, через которую проходят силовые линии.

Это не площадь самого провода, а площадь, которую он окружает.Это означает, что если вы согнете проволоку в круг, площадь, которую мы будем использовать при вычислении потока, будет площадью поверхности круга, а не проволоки.

На этом рисунке, где магнит находится в той же плоскости, что и контур цепи, не было бы тока, даже если бы магнит перемещался все ближе и дальше. Это связано с тем, что силовые линии магнитного поля не проходят через замкнутое пространство, а параллельны ему. Силовые линии магнитного поля должны проходить через область, ограниченную контуром цепи, чтобы возникла ЭДС.

Направление индуцированного тока (ESBQ2)

Самая важная вещь, которую следует помнить, это то, что индуцированный ток противодействует происходящему изменению.

На первом рисунке (слева) контурная петля имеет южный полюс приближающегося магнита. Величина поля от магнита становится больше. Реакция наведенной ЭДС будет состоять в том, чтобы попытаться противодействовать усилению поля по направлению к полюсу. Поле является вектором, поэтому ток будет течь в таком направлении, что поля, возникающие из-за тока, имеют тенденцию нейтрализовать поля от магнита, сохраняя результирующее поле неизменным.

Чтобы противостоять переходу от приближающегося южного полюса сверху, ток должен приводить к силовым линиям, которые удаляются от приближающегося полюса. Следовательно, индуцированное магнитное поле должно иметь силовые линии, идущие вниз внутри петли. Направление тока, указанное стрелками на контуре цепи, будет достигнуто. Проверьте это, используя Правило правой руки. Поместите большой палец правой руки в направлении одной из стрелок и обратите внимание на то, что поле закручивается вниз в область, ограниченную петлей.

На второй диаграмме южный полюс удаляется. Это означает, что поле от магнита станет слабее. Отклик на индуцированный ток будет заключаться в создании магнитного поля, которое добавляется к существующему от магнитного поля, чтобы противостоять его уменьшению в силе.

Другой способ представить ту же функцию — использовать полюса. Чтобы противостоять приближающемуся южному полюсу, индуцируемый ток создает поле, которое выглядит как другой южный полюс со стороны приближающегося южного полюса.Подобно отталкиванию полюсов, вы можете представить себе, как течение создает южный полюс, чтобы отразить приближающийся южный полюс. На второй панели ток устанавливает северный полюс, чтобы привлечь южный полюс и остановить его движение.

Мы также можем использовать вариант правила правой руки, помещая пальцы в направлении течения, чтобы большой палец указывал в направлении силовых линий (или северного полюса).

Мы можем проверить все это на случаях, когда северный полюс перемещается ближе или дальше от цепи.В первом случае приближения северного полюса ток будет сопротивляться изменению, создавая поле в направлении, противоположном полю, исходящему от магнита, который становится сильнее. Используйте Правило правой руки, чтобы убедиться, что стрелки создают поле с линиями поля, которые изгибаются вверх в замкнутой области, нейтрализуя те, которые изгибаются вниз от северного полюса магнита.

Подобно отталкиванию полюсов, в качестве альтернативы проверьте, что если поместить пальцы правой руки в направлении течения, большой палец будет указывать вверх, указывая на северный полюс.

Для второго рисунка, где северный полюс удаляется, ситуация обратная.

Направление индуцированного тока в соленоиде (ESBQ3)

Подход к изучению направления тока в соленоиде аналогичен подходу, описанному выше. Единственное отличие состоит в том, что в соленоиде есть несколько витков проволоки, поэтому величина наведенной ЭДС будет другой. Поток будет рассчитан с использованием площади поверхности соленоида, умноженной на количество петель.

Помните: направления токов и связанных с ними магнитных полей можно найти, используя только Правило правой руки. Когда пальцы правой руки направлены в направлении магнитного поля, большой палец указывает в направлении тока. Когда большой палец направлен в направлении магнитного поля, пальцы указывают в направлении тока.

Направление тока будет таким, чтобы препятствовать изменению. Мы бы использовали установку, как в этом скетче, для проведения теста:

В случае, когда северный полюс направлен к соленоиду, ток будет течь так, чтобы северный полюс установился на конце соленоида, ближайшем к приближающемуся магниту, чтобы оттолкнуть его (проверьте, используя Правило правой руки):

В случае, когда северный полюс движется от соленоида, ток будет течь так, что южный полюс будет установлен на конце соленоида, ближайшем к удаляющемуся магниту, чтобы притягивать его:

В случае, когда южный полюс движется от соленоида, ток будет течь так, что северный полюс будет установлен на конце соленоида, ближайшем к удаляющемуся магниту, чтобы притягивать его:

В случае, когда южный полюс направлен к соленоиду, ток будет течь так, что южный полюс будет установлен на конце соленоида, ближайшем к приближающемуся магниту, чтобы оттолкнуть его:

Простой способ создать магнитное поле изменяющейся интенсивности — переместить постоянный магнит рядом с проволокой или катушкой с проволокой.Магнитное поле должно увеличиваться или уменьшаться по напряженности перпендикулярно проводу (так, чтобы силовые линии магнитного поля «пересекали» проводник), иначе не будет индуцироваться напряжение.

Наведенный ток создает магнитное поле. Индуцированное магнитное поле имеет направление, которое стремится нейтрализовать изменение магнитного поля в петле из проволоки. Итак, вы можете использовать Правило правой руки, чтобы найти направление индуцированного тока, помня, что индуцированное магнитное поле противоположно направлению изменения магнитного поля.

Индукция

Электромагнитная индукция находит практическое применение в конструкции электрических генераторов, которые используют механическую энергию для перемещения магнитного поля мимо катушек с проволокой для генерации напряжения. Однако это далеко не единственное практическое применение этого принципа.

Если мы вспомним, магнитное поле, создаваемое проводом с током, всегда перпендикулярно проводу, и что сила потока этого магнитного поля зависит от величины тока, который проходит через него.Таким образом, мы можем видеть, что провод способен создавать напряжение на своей собственной длине , если ток изменяется. Этот эффект называется самоиндукцией . Самоиндукция — это когда изменяющееся магнитное поле создается изменением тока через провод, вызывая напряжение по длине того же провода.

Если магнитный поток усиливается путем сгибания проволоки в форме катушки и / или наматывания этой катушки на материал с высокой проницаемостью, этот эффект самоиндуцированного напряжения будет более интенсивным.Устройство, созданное для использования этого эффекта, называется индуктором .

Помните, что индуцированный ток создает магнитное поле, которое противодействует изменению магнитного потока. Это известно как закон Ленца.

Рабочий пример 1: закон Фарадея

Рассмотрим плоскую квадратную катушку с 5 витками. Катушка находится \ (\ text {0,50} \) \ (\ text {m} \) с каждой стороны и имеет магнитное поле \ (\ text {0,5} \) \ (\ text {T} \) проходя через него. Плоскость катушки перпендикулярна магнитному полю: поле направлено за пределы страницы.Используйте закон Фарадея для вычисления наведенной ЭДС, если магнитное поле увеличивается равномерно от \ (\ text {0,5} \) \ (\ text {T} \) до \ (\ text {1} \) \ (\ текст {T} \) в \ (\ text {10} \) \ (\ text {s} \). Определите направление индуцированного тока.

Определите, что требуется

Мы обязаны использовать Закон Фарадея для расчета наведенной ЭДС.

Запишите закон Фарадея

\ [\ mathcal {E} = — N \ frac {\ Delta \ phi} {\ Delta t} \] Мы знаем, что магнитное поле расположено под прямым углом к ​​поверхности и поэтому выровнено с нормалью.Это означает, что нам не нужно беспокоиться об угле, который поле образует с нормалью и \ (\ phi = BA \). Начальное или начальное магнитное поле, \ (B_i \), задается как конечная величина поля, \ (B_f \). Мы хотим определить величину ЭДС, чтобы можно было игнорировать знак минус.

Площадь \ (A \) — это площадь квадратной катушки. 2 (\ text {1} — \ text {0,50})} {\ text {10}} \\ & = \ текст {0,0625} \ текст {V} \ end {выровнять *}

Наведенный ток направлен против часовой стрелки, если смотреть со стороны нарастающего магнитного поля.

Рабочий пример 2: закон Фарадея

Рассмотрим соленоид из 9 витков с неизвестным радиусом \ (r \). На соленоид действует магнитное поле \ (\ text {0,12} \) \ (\ text {T} \). Ось соленоида параллельна магнитному полю. Когда поле равномерно переключается на \ (\ text {12} \) \ (\ text {T} \) в течение 2 минут, ЭДС величиной \ (- \ text {0,3} \) \ (\ text {V} \) индуцируется. Определите радиус соленоида.

Определите, что требуется

Требуется определить радиус соленоида.Мы знаем, что связь между наведенной ЭДС и полем регулируется законом Фарадея, который включает геометрию соленоида. Мы можем использовать это соотношение, чтобы найти радиус.

Запишите закон Фарадея

\ [\ mathcal {E} = — N \ frac {\ Delta \ phi} {\ Delta t} \] Мы знаем, что магнитное поле расположено под прямым углом к ​​поверхности и поэтому выровнено с нормалью. Это означает, что нам не нужно беспокоиться об угле, который поле образует с нормалью и \ (\ phi = BA \).{- \ text {2}} \) \ (\ text {m} \). На соленоид действует переменное магнитное поле, которое равномерно изменяется от \ (\ text {0,4} \) \ (\ text {T} \) до \ (\ text {3,4} \) \ (\ text { T} \) в интервале \ (\ text {27} \) \ (\ text {s} \). Ось соленоида составляет угол \ (\ text {35} \) \ (\ text {°} \) к магнитному полю. Найдите наведенную ЭДС.

Определите, что требуется

Мы обязаны использовать Закон Фарадея для расчета наведенной ЭДС.

Запишите закон Фарадея

\ [\ mathcal {E} = — N \ frac {\ Delta \ phi} {\ Delta t} \] Мы знаем, что магнитное поле расположено под углом к ​​нормали к поверхности.{- \ text {3}} \ text {V} \ end {выровнять *}

Наведенный ток направлен против часовой стрелки, если смотреть со стороны нарастающего магнитного поля.

Реальные приложения

Следующие устройства используют в своей работе закон Фарадея.

  • плиты индукционные

  • магнитофонов

  • металлоискатели

  • трансформаторы

Реальные применения закона Фарадея

Выберите одно из следующих устройств и поищите в Интернете или библиотеке, как работает ваше устройство.В объяснении вам нужно будет сослаться на закон Фарадея.

  • плиты индукционные

  • магнитофонов

  • металлоискатели

  • трансформаторы

Высокие оценки в науке — залог вашего успеха и будущих планов. Проверьте себя и узнайте больше о практике Сиявулы.

Зарегистрируйтесь и проверьте себя

Закон Фарадея

Учебник Упражнение 10.2

Изложите закон электромагнитной индукции Фарадея словами и запишите математическое соотношение.

ЭДС \ (\ mathcal {E} \), создаваемая вокруг контура проводника, пропорциональна скорости изменения магнитного потока φ через площадь A контура. Математически это можно выразить как:

\ [\ mathcal {E} = -N \ frac {\ Delta \ phi} {\ Delta t} \]

где \ (\ phi = B · A \), а B — напряженность магнитного поля.\ (N \) — количество контуров схемы. Магнитное поле измеряется в теслах (Тл). Знак минус указывает направление и то, что наведенная ЭДС имеет тенденцию противодействовать изменению магнитного потока. Знак минус можно игнорировать при вычислении звездных величин.

Опишите, что происходит, когда стержневой магнит вдавливается в соленоид, подключенный к амперметру, или вытягивается из него. Нарисуйте картинки, подтверждающие ваше описание.

В случае, когда северный полюс направлен к соленоиду, ток будет течь так, чтобы северный полюс установился на конце соленоида, ближайшем к приближающемуся магниту, чтобы оттолкнуть его (проверьте, используя Правило правой руки):

В случае, когда северный полюс движется от соленоида, ток будет течь так, что южный полюс будет установлен на конце соленоида, ближайшем к удаляющемуся магниту, чтобы притягивать его:

В случае, когда южный полюс движется от соленоида, ток будет течь так, что северный полюс будет установлен на конце соленоида, ближайшем к удаляющемуся магниту, чтобы притягивать его:

В случае, когда южный полюс направлен к соленоиду, ток будет течь так, что южный полюс будет установлен на конце соленоида, ближайшем к приближающемуся магниту, чтобы оттолкнуть его:

Объясните, как магнитный поток может быть равен нулю, когда магнитное поле не равно нулю.

Поток связан с магнитным полем:

\ (\ phi = BA \ cos \ theta \)

Если \ (\ cos \ theta \) равно 0, то магнитный поток будет равен 0, даже если есть магнитное поле. В этом случае магнитное поле параллельно поверхности и не проходит через нее.

Используйте правило правой руки, чтобы определить направление индуцированного тока в соленоиде ниже.

Южный полюс магнита приближается к соленоиду.Закон Ленца говорит нам, что ток будет течь, чтобы противодействовать изменению. Южный полюс на конце соленоида будет противодействовать приближающемуся южному полюсу. Ток будет циркулировать по странице в верхней части катушки, так что большой палец правой руки будет указывать влево.

Рассмотрим круговую катушку из 5 витков с радиусом \ (\ text {1,73} \) \ (\ text {m} \). Катушка подвергается воздействию переменного магнитного поля, которое равномерно изменяется от \ (\ text {2,18} \) \ (\ text {T} \) до \ (\ text {12,7} \) \ (\ text { T} \) в интервале \ (\ text {3} \) \ (\ text {minutes} \). {2} & = \ текст {0,0479} \\ г & = \ текст {0,22} \ текст {м} \ end {выровнять *}

Найдите изменение потока, если ЭДС равна \ (\ text {12} \) \ (\ text {V} \) за период \ (\ text {12} \) \ (\ text {s} \) .

\ begin {align *} \ mathcal {E} & = N \ frac {\ Delta \ phi} {\ Delta t} \\ 12 & = 5 \ left (\ frac {\ Delta \ phi} {12} \ right) \\ \ Delta \ phi & = \ text {28,8} \ text {Wb} \ end {выровнять *}

Если угол изменить на \ (\ text {45} \) \ (\ text {°} \), на какой временной интервал нужно изменить, чтобы наведенная ЭДС оставалась прежней?

\ begin {align *} \ mathcal {E} & = N \ frac {\ Delta \ phi} {\ Delta t} \\ & = N \ frac {\ phi_ {f} — \ phi_ {i}} {\ Delta t} \\ & = N \ frac {B_ {f} A \ cos \ theta — B_ {i} A \ cos \ theta} {\ Delta t} \\ & = \ cos \ theta \ times N \ frac {B_ {f} A — B_ {i} A} {\ Delta t} \ end {выровнять *}

Все значения остаются неизменными между двумя описанными ситуациями, за исключением угла и времени.Мы можем приравнять уравнения для двух сценариев:

\ begin {align *} \ mathcal {E} _1 & = \ mathcal {E} _2 \\ \ cos \ theta_1 \ times N \ frac {B_ {f} A — B_ {i} A} {\ Delta t_1} & = \ cos \ theta_2 \ times N \ frac {B_ {f} A — B_ {i} A } {\ Delta t_2} \\ \ cos \ theta_1 \ frac {1} {\ Delta t_1} & = \ cos \ theta_2 \ frac {1} {\ Delta t_2} \\ \ Delta t_2 & = \ frac {\ Delta t_1 \ cos \ theta_2} {\ cos \ theta_1} \\ \ Delta t_2 & = \ frac {(\ text {12} \ cos (\ text {45}} {\ cos (\ text {23})} \\ \ Delta t_2 & = \ text {9,22} \ text {s} \ end {выровнять *}

Закон Ленца — Университетская физика, том 2

Цели обучения

К концу этого раздела вы сможете:

  • Используйте закон Ленца для определения направления наведенной ЭДС при изменении магнитного потока
  • Используйте закон Фарадея с законом Ленца, чтобы определить наведенную ЭДС в катушке и соленоиде

Направление, в котором индуцированная ЭДС движет ток по проволочной петле, можно определить через отрицательный знак.Однако обычно это направление легче определить с помощью закона Ленца, названного в честь его первооткрывателя Генриха Ленца (1804–1865). (Фарадей также открыл этот закон, независимо от Ленца.) Мы формулируем закон Ленца следующим образом:

Закон Ленца

Направление индуцированной ЭДС направляет ток вокруг проволочной петли, чтобы всегда противодействовать изменению магнитного потока, вызывающему ЭДС.

Закон Ленца также можно рассматривать с точки зрения сохранения энергии. Если толкание магнита в катушку вызывает ток, энергия в этом токе должна исходить откуда-то.Если индуцированный ток вызывает магнитное поле, противодействующее увеличению поля магнита, который мы втолкнули, тогда ситуация ясна. Мы приложили магнит к полю и поработали с системой, и это проявилось как ток. Если бы индуцированное поле не противодействовало изменению магнитного потока, магнит был бы втянут, создавая ток без каких-либо действий. Была бы создана электрическая потенциальная энергия, нарушив закон сохранения энергии.

Чтобы определить наведенную ЭДС, вы сначала рассчитываете магнитный поток, а затем получаете Величину, заданную по формуле. Наконец, вы можете применить закон Ленца для определения значения.Это будет развиваться на примерах, которые иллюстрируют следующую стратегию решения проблем.

Стратегия решения проблем: закон Ленца

Чтобы использовать закон Ленца для определения направлений индуцированных магнитных полей, токов и ЭДС:

  1. Сделайте набросок ситуации для использования при визуализации и записи направлений.
  2. Определите направление приложенного магнитного поля
  3. Определите, увеличивается или уменьшается его магнитный поток.
  4. Теперь определите направление индуцированного магнитного поля. Индуцированное магнитное поле пытается усилить магнитный поток, который уменьшается, или противодействует магнитному потоку, который увеличивается. Следовательно, индуцированное магнитное поле добавляет или вычитает приложенное магнитное поле, в зависимости от изменения магнитного потока.
  5. Используйте правило правой руки 2 (RHR-2; см. Магнитные силы и поля), чтобы определить направление индуцированного тока I , ответственного за индуцированное магнитное поле
  6. Направление (или полярность) наведенной ЭДС теперь может управлять обычным током в этом направлении.

Применим закон Ленца к системе (Рисунок) (а). Мы обозначаем «перед» замкнутой проводящей петли как область, содержащую приближающийся стержневой магнит, а «заднюю часть» петли как другую область. Когда северный полюс магнита движется к петле, поток через петлю из-за поля магнита увеличивается, потому что сила силовых линий, направленных от передней части петли к задней, увеличивается. Поэтому в контуре индуцируется ток. По закону Ленца направление индуцированного тока должно быть таким, чтобы его собственное магнитное поле было направлено таким образом, чтобы противостояло изменяющемуся потоку, вызванному полем приближающегося магнита.Следовательно, индуцированный ток циркулирует так, что силовые линии его магнитного поля через петлю направлены от задней части петли к передней. При использовании RHR-2 поместите большой палец напротив силовых линий магнитного поля, то есть к стержневому магниту. Ваши пальцы сгибаются против часовой стрелки, если смотреть со стороны стержневого магнита. В качестве альтернативы, мы можем определить направление индуцированного тока, рассматривая токовую петлю как электромагнит, который противодействует приближению северного полюса стержневого магнита.Это происходит, когда индуцированный ток течет, как показано, поскольку тогда поверхность петли ближе к приближающемуся магниту также является северным полюсом.

Изменение магнитного потока, вызванное приближением магнита, индуцирует ток в контуре. (а) Приближающийся северный полюс индуцирует ток против часовой стрелки по отношению к стержневому магниту. (b) Приближающийся южный полюс индуцирует ток по часовой стрелке относительно стержневого магнита.

На части (b) рисунка показан южный полюс магнита, движущийся к проводящей петле.В этом случае поток через петлю из-за поля магнита увеличивается, потому что количество силовых линий, направленных от задней части петли к передней, увеличивается. Чтобы противодействовать этому изменению, в петле индуцируется ток, силовые линии которого через петлю направлены спереди назад. Точно так же мы можем сказать, что ток течет в таком направлении, что поверхность петли ближе к приближающемуся магниту является южным полюсом, который затем отталкивает приближающийся южный полюс магнита.При использовании RHR-2 ваш большой палец направлен в сторону от стержневого магнита. Ваши пальцы сгибаются по часовой стрелке, что соответствует направлению индуцированного тока.

Другой пример, иллюстрирующий использование закона Ленца, показан на (Рисунок). Когда переключатель разомкнут, уменьшение тока через соленоид вызывает уменьшение магнитного потока через его катушки, что вызывает ЭДС в соленоиде. Эта ЭДС должна противодействовать вызывающему его изменению (прекращению тока). Следовательно, наведенная ЭДС имеет указанную полярность и движется в направлении исходного тока.Это может вызвать дугу на выводах переключателя при его размыкании.

(а) Соленоид, подключенный к источнику ЭДС. (b) Размыкающий переключатель S прекращает подачу тока, что, в свою очередь, вызывает ЭДС в соленоиде. (c) Разность потенциалов между концами заостренных стержней создается за счет индукции ЭДС в катушке. Эта разность потенциалов достаточно велика, чтобы образовалась дуга между острыми точками.

Проверьте свое понимание Найдите направление индуцированного тока в проводной петле, показанной ниже, когда магнит входит, проходит и покидает петлю.

Для показанного наблюдателя ток течет по часовой стрелке по мере приближения магнита, уменьшается до нуля, когда магнит центрируется в плоскости катушки, а затем течет против часовой стрелки, когда магнит покидает катушку.

Проверьте свое понимание Проверьте направления наведенных токов на (рисунок).

Сводка

  • Мы можем использовать закон Ленца для определения направлений индуцированных магнитных полей, токов и ЭДС.
  • Направление наведенной ЭДС всегда противодействует изменению магнитного потока, вызывающему ЭДС, результат, известный как закон Ленца.

Концептуальные вопросы

Круглые токопроводящие петли, показанные на прилагаемом рисунке, параллельны, перпендикулярны плоскости страницы и соосны. (a) Когда переключатель S замкнут, в каком направлении индуцируется ток в D ? (b) Когда переключатель разомкнут, какое направление тока индуцируется в контуре D ?

а.CW со стороны схемы; б. Против часовой стрелки, если смотреть со стороны схемы

Северный полюс магнита перемещается к медной петле, как показано ниже. Если вы смотрите на петлю сверху магнита, скажете ли вы, что индуцированный ток циркулирует по или против часовой стрелки?

На прилагаемом рисунке показано проводящее кольцо в различных положениях при его движении в магнитном поле. В чем смысл индуцированной ЭДС для каждой из этих позиций?

При входе в петлю наведенная ЭДС создает ток против часовой стрелки, а при выходе из петли индуцированная ЭДС создает непрерывный ток.В то время как петля полностью находится внутри магнитного поля, нет изменения потока и, следовательно, нет индуцированного тока.

Покажите, что и у вас такие же единицы.

Укажите направление индуцированного тока для каждого случая, показанного ниже, наблюдая со стороны магнита.

а. Против часовой стрелки, если смотреть со стороны магнита; б. CW, если смотреть со стороны магнита; c. CW, если смотреть со стороны магнита; d. Против часовой стрелки, если смотреть со стороны магнита; е. CW, если смотреть со стороны магнита; f. нет тока

Проблемы

Одновитковая круглая петля из проволоки радиусом 50 мм расположена в плоскости, перпендикулярной пространственно однородному магнитному полю.За интервал времени 0,10 с величина поля равномерно увеличивается от 200 до 300 мТл. (а) Определите ЭДС, наведенную в петле. (б) Если магнитное поле направлено за пределы страницы, каково направление тока, индуцируемого в петле?

а. ; б. Против часовой стрелки с той же точки зрения, что и магнитное поле

При первом включении магнитного поля поток через 20-витковую петлю изменяется со временем в зависимости от того, где он находится в милливеберах, t — в секундах, и петля находится в плоскости страницы с нормальным направлением устройства. наружу.(а) Какая ЭДС индуцируется в контуре как функция времени? Каково направление индуцированного тока при (б) t = 0, (в) 0,10, (г) 1,0 и (д) 2,0 с?

а. 150 А вниз через резистор; б. 46 А вверх через резистор; c. 0,019 А вниз через резистор

Используйте закон Ленца для определения направления индуцированного тока в каждом случае.

Глоссарий

Закон Ленца
направление наведенной ЭДС противодействует изменению магнитного потока, который ее произвел; это отрицательный знак в законе Фарадея

20.3 Электромагнитная индукция — физика

Изменение магнитных полей

В предыдущем разделе мы узнали, что ток создает магнитное поле. Если природа симметрична, то, возможно, магнитное поле может создать ток. В 1831 году, примерно через 12 лет после открытия, что электрический ток создает магнитное поле, английский ученый Майкл Фарадей (1791–1862) и американский ученый Джозеф Генри (1797–1878) независимо друг от друга продемонстрировали, что магнитные поля могут создавать токи.Основной процесс генерации токов с помощью магнитных полей называется индукцией; этот процесс также называют магнитной индукцией, чтобы отличить его от индукционной зарядки, в которой используется электростатическая кулоновская сила.

Когда Фарадей открыл то, что сейчас называется законом индукции Фарадея, королева Виктория спросила его, как можно использовать электричество. «Мадам, — ответил он, — что хорошего в ребенке?» Сегодня токи, индуцированные магнитными полями, необходимы нашему технологическому обществу. Электрический генератор, который можно найти во всем, от автомобилей до велосипедов и атомных электростанций, использует магнетизм для генерации электрического тока.Другие устройства, которые используют магнетизм для индукции токов, включают в себя звукосниматели в электрогитарах, трансформаторы любого размера, определенные микрофоны, ворота безопасности аэропорта и механизмы демпфирования на чувствительных химических весах.

Один из экспериментов Фарадея для демонстрации магнитной индукции заключался в перемещении стержневого магнита через проволочную катушку и измерении результирующего электрического тока через проволоку. Схема этого эксперимента показана на рис. 20.33. Он обнаружил, что ток индуцируется только тогда, когда магнит движется относительно катушки.Когда магнит неподвижен по отношению к катушке, в катушке не индуцируется ток, как показано на рисунке 20.33. Кроме того, перемещение магнита в противоположном направлении (сравните рис. 20.33 с рис. 20.33) или изменение полярности магнита (сравните рис. 20.33 с рис. 20.33) приводит к возникновению тока в противоположном направлении.

Рис. 20.33 Движение магнита относительно катушки создает электрические токи, как показано. Такие же токи возникают, если катушку перемещать относительно магнита.Чем больше скорость, тем больше величина тока, и ток равен нулю, когда нет движения. Ток, возникающий при перемещении магнита вверх, имеет направление, противоположное направлению тока, возникающего при перемещении магнита вниз.

Виртуальная физика

Закон Фарадея

Попробуйте это моделирование, чтобы увидеть, как движение магнита создает ток в цепи. Лампочка загорается, чтобы показать, когда течет ток, а вольтметр показывает падение напряжения на лампочке.Попробуйте переместить магнит через четырехвитковую катушку и через двухвитковую катушку. Какая катушка производит более высокое напряжение при одинаковой скорости магнита?

Проверка захвата

Если северный полюс находится влево и магнит перемещается справа налево, при входе магнита в катушку создается положительное напряжение. Какое знаковое напряжение получится, если эксперимент повторить с южным полюсом слева?

  1. Знак напряжения изменится, потому что направление тока изменится при перемещении южного полюса магнита влево.
  2. Знак напряжения останется прежним, потому что направление тока не изменится при перемещении южного полюса магнита влево.
  3. Знак напряжения изменится, потому что величина протекающего тока изменится при перемещении южного полюса магнита влево.
  4. Знак напряжения останется прежним, потому что величина тока не изменится при перемещении южного полюса магнита влево.

Индуцированная электродвижущая сила

Если в катушке индуцируется ток, Фарадей рассуждал, что должно быть то, что он назвал электродвижущей силой , проталкивающей заряды через катушку. Эта интерпретация оказалась неверной; вместо этого внешний источник, выполняющий работу по перемещению магнита, добавляет энергию зарядам в катушке. Энергия, добавляемая на единицу заряда, измеряется в вольтах, поэтому электродвижущая сила на самом деле является потенциалом. К сожалению, название «электродвижущая сила» прижилось, а вместе с ним и возможность спутать его с реальной силой.По этой причине мы избегаем термина электродвижущая сила и просто используем сокращение эдс , которое имеет математический символ ε.ε. ЭДС может быть определена как скорость, с которой энергия отбирается от источника на единицу тока, протекающего по цепи. Таким образом, ЭДС — это энергия на единицу заряда , добавленная источником, которая контрастирует с напряжением, которое представляет собой энергию на единицу заряда , высвобождаемую , когда заряды проходят через цепь.

Чтобы понять, почему в катушке возникает ЭДС из-за движущегося магнита, рассмотрим рисунок 20.34, на котором показан стержневой магнит, движущийся вниз относительно проволочной петли. Первоначально через петлю проходят семь силовых линий магнитного поля (см. Изображение слева). Поскольку магнит удаляется от катушки, только пять силовых линий магнитного поля проходят через петлю за короткое время ΔtΔt (см. Изображение справа). Таким образом, когда происходит изменение количества силовых линий магнитного поля, проходящих через область, определяемую проволочной петлей, в проволочной петле индуцируется ЭДС. Подобные эксперименты показывают, что наведенная ЭДС пропорциональна скорости изменения магнитного поля.Математически мы выражаем это как

ε∝ΔBΔt, ε∝ΔBΔt,

20,24

где ΔBΔB — изменение величины магнитного поля за время ΔtΔt, а A — площадь петли.

Рис. 20.34 Стержневой магнит движется вниз относительно проволочной петли, так что количество силовых линий магнитного поля, проходящих через петлю, со временем уменьшается. Это вызывает в контуре ЭДС, создающую электрический ток.

Обратите внимание, что силовые линии магнитного поля, которые лежат в плоскости проволочной петли, на самом деле не проходят через петлю, как показано крайним левым витком на рисунке 20.35. На этом рисунке стрелка, выходящая из петли, представляет собой вектор, величина которого равна площади петли, а направление перпендикулярно плоскости петли. На рисунке 20.35 петля повернута от θ = 90 ° θ = 90 °. до θ = 0 °, θ = 0 ° вклад силовых линий магнитного поля в ЭДС увеличивается. Таким образом, для создания ЭДС в проволочной петле важна составляющая магнитного поля, которая находится на перпендикулярно плоскости петли, то есть Bcosθ.Bcosθ.

Это аналог паруса на ветру.Представьте, что проводящая петля — это парус, а магнитное поле — как ветер. Чтобы максимизировать силу ветра на парусе, парус ориентируют так, чтобы вектор его поверхности указывал в том же направлении, что и ветер, как в самой правой петле на рис. 20.35. Когда парус выровнен так, что его вектор поверхности перпендикулярен ветру, как в крайней левой петле на рис. 20.35, тогда ветер не оказывает силы на парус.

Таким образом, с учетом угла наклона магнитного поля к площади, пропорциональность E∝ΔB / ΔtE∝ΔB / Δt становится равной

E∝ΔBcosθΔt.E∝ΔBcosθΔt.

20,25

Рис. 20.35 Магнитное поле лежит в плоскости крайней левой петли, поэтому в этом случае оно не может генерировать ЭДС. Когда петля поворачивается так, что угол магнитного поля с вектором, перпендикулярным области петли, увеличивается до 90 ° 90 ° (см. Крайнюю правую петлю), магнитное поле вносит максимальный вклад в ЭДС в петле. Точки показывают, где силовые линии магнитного поля пересекают плоскость, определяемую петлей.

Другой способ уменьшить количество силовых линий магнитного поля, проходящих через проводящую петлю на Рисунке 20.35 не для перемещения магнита, а для уменьшения размера петли. Эксперименты показывают, что изменение площади проводящей петли в стабильном магнитном поле вызывает в петле ЭДС. Таким образом, ЭДС, создаваемая в проводящей петле, пропорциональна скорости изменения произведения перпендикулярного магнитного поля и площади петли

. ε∝Δ [(Bcosθ) A] Δt, ε∝Δ [(Bcosθ) A] Δt,

20,26

, где BcosθBcosθ — перпендикулярное магнитное поле, а A — площадь контура.Продукт BAcosθBAcosθ очень важен. Оно пропорционально количеству силовых линий магнитного поля, которые проходят перпендикулярно через поверхность площадью A . Возвращаясь к нашей аналогии с парусом, он будет пропорционален силе ветра на парусе. Он называется магнитным потоком и обозначается как ΦΦ.

Φ = BAcosθΦ = BAcosθ

20,27

Единицей измерения магнитного потока является Вебер (Вб), то есть магнитное поле на единицу площади, или Т / м 2 . Вебер — это также вольт-секунда (Vs).

Индуцированная ЭДС фактически пропорциональна скорости изменения магнитного потока через проводящую петлю.

ε∝ΔΦΔtε∝ΔΦΔt

20,28

Наконец, для катушки, изготовленной из петель Н , ЭДС в Н в раз сильнее, чем для одиночной петли. Таким образом, наведенная изменяющимся магнитным полем ЭДС в катушке из петель N составляет

ε∝NΔBcosθΔtA.ε∝NΔBcosθΔtA.

Последний вопрос, на который нужно ответить, прежде чем мы сможем преобразовать пропорциональность в уравнение: «В каком направлении течет ток?» Русский ученый Генрих Ленц (1804–1865) объяснил, что ток течет в том направлении, которое создает магнитное поле, которое пытается сохранить постоянный поток в контуре.Например, снова рассмотрим рисунок 20.34. Движение стержневого магнита приводит к уменьшению количества направленных вверх силовых линий магнитного поля, проходящих через петлю. Следовательно, в контуре генерируется ЭДС, которая направляет ток в направлении, которое создает более направленные вверх силовые линии магнитного поля. Используя правило правой руки, мы видим, что этот ток должен течь в направлении, показанном на рисунке. Чтобы выразить тот факт, что наведенная ЭДС действует как противодействие изменению магнитного потока через проволочную петлю, в пропорциональность ε∝ΔΦ / Δt вводится знак минус.) внутри катушки, направленной влево. Это будет противодействовать увеличению магнитного потока, направленного вправо. Чтобы увидеть, в каком направлении должен течь ток, направьте большой палец правой руки в желаемом направлении магнитного поля B → катушка, B → катушка, и ток будет течь в направлении, указанном сгибанием ваших пальцев правой руки. Это показано изображением правой руки в верхнем ряду рисунка 20.36. Таким образом, ток должен течь в направлении, показанном на рисунке 4 (а).

На Рисунке 4 (b) направление движения магнита изменено на обратное.В катушке направленное вправо магнитное поле B → magB → mag из-за движущегося магнита уменьшается. Закон Ленца гласит, что, чтобы противостоять этому уменьшению, ЭДС будет управлять током, который создает дополнительное направленное вправо магнитное поле B → катушка B → катушка в катушке. Опять же, направьте большой палец правой руки в желаемом направлении магнитного поля, и ток будет течь в направлении, указанном сгибанием ваших пальцев правой руки (рис. 4 (b)).

Наконец, на Рисунке 4 (c) магнит перевернут, так что южный полюс находится ближе всего к катушке.Теперь магнитное поле B → magB → mag направлено на магнит, а не на катушку. Когда магнит приближается к катушке, он вызывает увеличение направленного влево магнитного поля в катушке. Закон Ленца гласит, что ЭДС, индуцированная в катушке, будет управлять током в направлении, которое создает магнитное поле, направленное вправо. Это будет противодействовать увеличению магнитного потока, направленного влево из-за магнита. Повторное использование правила правой руки, как показано на рисунке, показывает, что ток должен течь в направлении, показанном на рисунке 4 (c).

Рис. 20.36. Закон Ленца гласит, что ЭДС, индуцированная магнитным полем, будет управлять током, который сопротивляется изменению магнитного потока в цепи. Это показано на панелях (а) — (с) для различных ориентаций и скоростей магнита. Правые руки справа показывают, как применить правило правой руки, чтобы найти, в каком направлении наведенный ток течет вокруг катушки.

Виртуальная физика

Электромагнитная лаборатория Фарадея

Это моделирование предлагает несколько действий.А пока щелкните вкладку Pickup Coil, которая представляет собой стержневой магнит, который вы можете перемещать через катушку. Когда вы это сделаете, вы увидите, как электроны движутся в катушке, и загорится лампочка, или вольтметр покажет напряжение на резисторе. Обратите внимание, что вольтметр позволяет вам видеть знак напряжения при перемещении магнита. Вы также можете оставить стержневой магнит в покое и переместить катушку, хотя наблюдать за результатами сложнее.

Проверка захвата

Сориентируйте стержневой магнит так, чтобы северный полюс был направлен вправо, и поместите приемную катушку справа от стержневого магнита.Теперь переместите стержневой магнит к катушке и посмотрите, в каком направлении движутся электроны. Это такая же ситуация, как показано ниже. Ток при моделировании течет в том же направлении, что и показано ниже? Объясните, почему да или почему нет.
  1. Да, ток в моделировании течет, как показано, потому что направление тока противоположно направлению потока электронов.
  2. Нет, ток в моделировании течет в противоположном направлении, потому что направление тока совпадает с направлением потока электронов.

Watch Physics

Наведенный ток в проводе

В этом видео объясняется, как можно индуцировать ток в прямом проводе, перемещая его через магнитное поле. Лектор использует перекрестное произведение , тип векторного умножения. Не волнуйтесь, если вы не знакомы с этим, он в основном объединяет правило правой руки для определения силы, действующей на заряды в проводе, с уравнением F = qvBsinθ.F = qvBsinθ.

Проверка захвата

Какая ЭДС создается по прямому проводу 0.через однородное магнитное поле (0,30 Тл) ? Провод лежит в направлении ŷ . Кроме того, какой конец провода находится под более высоким потенциалом — пусть нижний конец провода находится под углом y = 0, а верхний конец — под углом y = 0,5 м)?

  1. 0,15 В и нижний конец провода будет иметь более высокий потенциал
  2. 0,15 В и верхний конец провода будет иметь более высокий потенциал
  3. 0,075 В и нижний конец провода будет иметь более высокий потенциал
  4. 0.075 В и на верхнем конце провода будет более высокий потенциал

Рабочий пример

ЭДС, индуцированная в проводящей катушке движущимся магнитом

Представьте, что магнитное поле проходит через катушку в направлении, указанном на рисунке 20.37. Диаметр катушки 2,0 см. Если магнитное поле изменится с 0,020 до 0,010 Тл за 34 с, каковы направление и величина индуцированного тока? Предположим, что катушка имеет сопротивление 0,1 Ом.

Рисунок 20.37 Катушка, через которую проходит магнитное поле B .

Стратегия

Используйте уравнение ε = −NΔΦ / Δtε = −NΔΦ / Δt, чтобы найти наведенную ЭДС в катушке, где Δt = 34sΔt = 34s. Подсчитав количество витков соленоида, мы находим, что у него 16 петель, поэтому N = 16.N = 16. Используйте уравнение Φ = BAcosθΦ = BAcosθ для расчета магнитного потока

Φ = BAcosθ = Bπ (d2) 2, Φ = BAcosθ = Bπ (d2) 2,

20,30

, где d — диаметр соленоида, а мы использовали cos0 ° = 1. cos0 ° = 1. Поскольку площадь соленоида не меняется, изменение магнитного потока, проходящего через соленоид, составляет

ΔΦ = ΔBπ (d2) 2.ΔΦ = ΔBπ (d2) 2.

20,31

Найдя ЭДС, мы можем использовать закон Ома, ε = IR, ε = IR, чтобы найти ток.

Наконец, закон Ленца гласит, что ток должен создавать магнитное поле, которое препятствует уменьшению приложенного магнитного поля. Таким образом, ток должен создавать магнитное поле справа.

Решение

Объединение уравнений ε = −NΔΦ / Δtε = −NΔΦ / Δt и Φ = BAcosθΦ = BAcosθ дает

ε = −NΔΦΔt = −NΔBπd24Δt.ε = −NΔΦΔt = −NΔBπd24Δt.

20,32

Решение закона Ома для тока и использование этого результата дает

I = εR = −NΔBπd24RΔt = −16 (−0,010T) π (0,020 м) 24 (0,10 Ом) (34 с) = 15 мкА.I = εR = −NΔBπd24RΔt = −16 (−0,010T) π (0,020 м) 24 (0,10 Ом) (34 с) = 15 мкА.

20.33

Закон Ленца гласит, что ток должен создавать магнитное поле справа. Таким образом, мы направляем большой палец правой руки вправо и сжимаем пальцы правой руки вокруг соленоида. Ток должен течь в том направлении, в котором указывают наши пальцы, поэтому он входит в левый конец соленоида и выходит из правого конца.

Обсуждение

Давайте посмотрим, имеет ли смысл знак минус в законе индукции Фарадея. Определите направление магнитного поля как положительное. Это означает, что изменение магнитного поля отрицательное, как мы обнаружили выше. Знак минус в законе индукции Фарадея отрицает отрицательное изменение магнитного поля, оставляя нам положительный ток. Следовательно, ток должен течь в направлении магнитного поля, что мы и обнаружили.

Теперь попробуйте определить положительное направление как направление, противоположное направлению магнитного поля, то есть положительное направление находится слева на рисунке 20.37. В этом случае вы обнаружите отрицательный ток. Но поскольку положительное направление находится влево, отрицательный ток должен течь вправо, что снова согласуется с тем, что мы обнаружили с помощью закона Ленца.

Рабочий пример

Магнитная индукция из-за изменения размера цепи

Схема, показанная на рисунке 20.38, состоит из U-образного провода с резистором, концы которого соединены скользящим токопроводящим стержнем. Магнитное поле, заполняющее область, ограниченную контуром, имеет постоянное значение 0.01 T. Если стержень тянут вправо со скоростью v = 0,50 м / с, v = 0,50 м / с, какой ток индуцируется в цепи и в каком направлении он течет?

Рисунок 20.38 Схема ползунка. Магнитное поле постоянно, и шток тянется вправо со скоростью v . Изменяющаяся область, заключенная в цепи, вызывает в цепи ЭДС.

Стратегия

Мы снова используем закон индукции Фарадея, E = −NΔΦΔt, E = −NΔΦΔt, хотя на этот раз магнитное поле остается постоянным и площадь, ограниченная контуром, изменяется.Схема состоит из одного контура, поэтому N = 1.N = 1. Скорость изменения площади ΔAΔt = vℓ.ΔAΔt = vℓ. Таким образом, скорость изменения магнитного потока составляет

ΔΦΔt = Δ (BAcosθ) Δt = BΔAΔt = Bvℓ, ΔΦΔt = Δ (BAcosθ) Δt = BΔAΔt = Bvℓ,

20,34

, где мы использовали тот факт, что угол θθ между вектором площади и магнитным полем равен 0 °. Зная ЭДС, мы можем найти ток, используя закон Ома. Чтобы найти направление тока, мы применяем закон Ленца.

Решение

Закон индукции Фарадея дает

E = −NΔΦΔt = −Bvℓ.E = −NΔΦΔt = −Bvℓ.

20,35

Решение закона Ома для тока и использование предыдущего результата для ЭДС дает

I = ER = −BvℓR = — (0,010T) (0,50 м / с) (0,10 м) 20Ω = 25 мкA I = ER = −BvℓR = — (0,010T) (0,50 м / с) (0,10 м) 20Ω = 25 мкА.

20,36

По мере того, как стержень скользит вправо, магнитный поток, проходящий через контур, увеличивается. Закон Ленца говорит нам, что индуцированный ток создаст магнитное поле, которое будет противодействовать этому увеличению. Таким образом, магнитное поле, создаваемое индуцированным током, должно попадать на страницу.Сгибание петли пальцами правой руки по часовой стрелке заставляет большой палец правой руки указывать на страницу, что является желаемым направлением магнитного поля. Таким образом, ток должен течь по цепи по часовой стрелке.

Обсуждение

Сохраняется ли энергия в этой цепи? Внешний агент должен тянуть стержень с достаточной силой, чтобы просто уравновесить силу на проводе с током в магнитном поле — вспомните, что F = IℓBsinθ.F = IℓBsinθ. Скорость, с которой эта сила действует на стержень, должна уравновешиваться скоростью, с которой цепь рассеивает мощность.Используя F = IℓBsinθ, F = IℓBsinθ, сила, необходимая для протягивания проволоки с постоянной скоростью v , равна

. Fpull = IℓBsinθ = IℓB, Fpull = IℓBsinθ = IℓB,

20,37

, где мы использовали тот факт, что угол θθ между током и магнитным полем составляет 90 ° 0,90 °. Подставляя приведенное выше выражение для тока в это уравнение, получаем

Fpull = IℓB = −BvℓR (ℓB) = — B2vℓ2R.Fpull = IℓB = −BvℓR (ℓB) = — B2vℓ2R.

20,38

Сила, создаваемая агентом, тянущим стержень, равна Fpullv, или Fpullv, или

. Потяните = Fpullv = −B2v2ℓ2R.Потяните = Fpullv = −B2v2ℓ2R.

20,39

Мощность, рассеиваемая схемой, составляет

Pdissipated = I2R = (- BvℓR) 2R = B2v2ℓ2R. Pdissipated = I2R = (- BvℓR) 2R = B2v2ℓ2R.

20,40

Таким образом, мы видим, что Ppull + Pdissipated = 0, Ppull + Pdissipated = 0, что означает, что мощность сохраняется в системе, состоящей из цепи и агента, который тянет стержень. Таким образом, в этой системе сохраняется энергия.

закон Фарадея

закон Фарадея

Закон Фарадея

Закон Фарадея — одно из уравнений Максвелла.Закон Фарадея гласит, что абсолютная величина или величина обращения электрическое поле E вокруг замкнутого контура равно скорости изменения магнитный поток через область, ограниченную петлей. В приведенное ниже уравнение выражает закон Фарадея в математической форме.

ΔΦ B / Δt (через фиксированная площадь) = -Σ вокруг контура E ∙ r (при фиксированное время)

Знак минус в этом уравнении говорит нам о направлении тираж.(См. Ниже.)

Когда магнитный поток через замкнутую область при изменении петли Σ вокруг петли E ∙ r не равно нулю, электрическое поле E циркулирует.
E ∙
r — работа, выполненная за единичный заряд электрическим полем при перемещении заряда на расстояние ∆ r .
Если петля — это настоящая проволочная петля, тогда есть фактическая работа, выполняемая индуцированным поле на бесплатные начисления.
Σ вокруг петли E ∙ r — работа на единицу заряда полем при однократном перемещении заряда по петле.
Это наведенная ЭДС , и измеряется в вольтах.
Индуцированная ЭДС вызывает протекание тока без разность потенциалов из-за разделенных зарядов.

ΔΦ B / Δt (через фиксированная площадь) = наведенная ЭДС

Индуцированное электрическое поле НЕ консервативное поле.Когда вы перемещаете заряд против индуцированного поле один раз по кругу, вам нужно работать. Но твоя работа НЕ хранится как потенциальная энергия. Вы не можете позволить электрическому полю работать, чтобы восстановиться энергия, которую вы потратили на перемещение заряда. Индуцированное электрическое поле исчезает как как только магнитный поток перестанет меняться. Работа, которую ты делаешь на заряд против индуцированного поля не локально хранится. Энергия может быть отведена в виде электромагнитная волна. Электромагнитные волны переносят энергию через свободное пространство.

Какое направление динамического (индуцированного) поля?

Знак минус в уравнении, выражающем закон Фарадея, говорит нам о направление индуцированного поля.
Есть простой способ запомнить это направление. Циркуляция индуцированного поля равна ЭДС.
Любой текущий течет в результате этой ЭДС создает магнитное поле, которое противодействует изменения потока, которые его производят.
Это называется Закон Ленца.

Индуцированная ЭДС действует как противодействие изменению потока, которое произвести это.

Пример:

Магнит быстро перемещается к проволочной петле, как показано.
Поток через проволочную петлю увеличивается в направлении вниз.
Ток начинает течь в петлю в направлении, указанном стрелкой.
Магнитное поле, создаваемое этим током указывает вверх, противостоит потоку изменения, которые его производят.
Магнитная сила из-за петли на магните действует, чтобы замедлить приближающийся магнит.

Прелесть закона Ленца в том, что вам не нужно вдаваться в подробности. Если магнитный поток через проводник изменяется, токи будут течь встречно что бы ни вызвало изменение. Если какое-то относительное движение вызывает изменение потока, ток попытается остановить это относительное движение. Если изменение тока в цепь отвечает за изменение потока, тогда наведенная ЭДС будет пытаться предотвратить изменение тока в этой цепи.

Обратите внимание: Электромагнитная индукция и закон Фарадея (Youtube)

Проблема:

Рассмотрим плоскую квадратную катушку с N = 5 витками.
Катушка имеет длину 20 см с каждой стороны и имеет магнитное поле. 0,3 Тл.
Плоскость катушки перпендикулярна плоскости магнитное поле: поле указывает за пределы страницы.
а) Если ничего не изменилось, какова наведенная ЭДС?
(б) Магнитное поле равномерно увеличивается от 0.От 3 до 0,8 Тл за 1 с. Какова наведенная ЭДС в катушке, пока происходит изменение?
(c) При изменении магнитного поля ЭДС, индуцированная в катушке, вызывает ток течь. Ток течет по часовой стрелке или против часовой стрелки? вокруг катушки?

Решение:

  • Рассуждение:
    Если величина магнитного поля B изменяется, то поток Φ = BA изменяется, и возникает ЭДС.
  • Детали расчета:
    (a) ЭДС индуцируется изменяющимся магнитным потоком.Если ничего изменяется, наведенная ЭДС равна нулю.
    (б) Катушка имеет 5 витков. Каждый поворот имеет площадь A = (0,2 м) 2 . Начальный магнитный поток через каждый оборот катушки Φ 0 = B 0 A = 0,3 * (0,2) 2 Tm 2 = 0,012 Tm 2 .
    Конечный магнитный поток через каждый виток катушки Φ f = B f A = 0,8 * (0,2) 2 Tm 2 = 0,032 Tm 2 .
    Суммарное изменение потока через катушку N (Φ ф — Φ 0 ), с N = 5. Индуцированная ЭДС составляет
    ЭДС = -N∆Φ / ∆t = -N (Φ f — Φ 0 ) / ∆t = [-5 * (0,032 -0,012) /1,0] V = -0,1 В.
    (c) При изменении магнитного поля магнитный поток увеличивался. со страницы. По закону Ленца наведенная в петле ЭДС этим изменяющимся потоком образуется ток, который создает поле, противодействующее изменять.Поле, создаваемое током в катушке, указывает на страницу, противоположную направлению увеличения потока. Чтобы произвести поле на страницу, ток должен течь по кругу по часовой стрелке. согласно правилу правой руки.

Модуль 5: Вопрос 1

Стержневой магнит расположен перед горизонтальной проволочной петлей с его северный полюс, указывающий на петлю. Затем магнит отрывается от петля. Идет ли индуцированный ток в контуре по часовой стрелке или против часовой стрелки?

Обсудите это со своими однокурсниками на дискуссионном форуме!
Визуализируйте магнитное поле стержневого магнита.Как происходит поток этого поле через проводную петлю поменять?


Самоиндукция

Если длинная катушка провода сечением A и длиной ℓ с N витками подключен или отключен от батареи, изменение магнитного потока через катушка производит наведенную ЭДС. Индуцированный ток создает магнитное поле, которое противодействует изменению магнитного потока. Величина наведенная ЭДС может быть рассчитана с помощью закона Фарадея.

  • Магнитное поле внутри длинной катушки B = μ 0 (N / ℓ) I.
  • Поток через катушку равен NBA = μ 0 (N 2 /) IA.
  • Изменение потока в единицу времени составляет μ 0 (N 2 / ℓ) A ∆I / ∆t = L * ∆I / ∆t, поскольку I — единственная величина меняется со временем.
    L = μ 0 (N 2 / ℓ) A называется собственная индуктивность катушки. В единицы индуктивности — Генри (Гн) .1 H = 1 Вс / А.
  • Индуцированная ЭДС равна ЭДС = -L * ∆I / ∆t, где знак минус является следствием закона Ленца.

Индуцированная ЭДС пропорциональна скорости изменения тока в катушка. Оно может быть в несколько раз больше напряжения источника питания. Когда выключатель в цепи, по которой проходит большой ток, размыкается, уменьшая ток до ноль за очень короткий промежуток времени, это может привести к искре. Все схемы имеют собственную индуктивность, и у нас всегда есть ЭДС = -L * ∆I / ∆t.Собственная индуктивность L зависит только от по геометрии схемы.

Проблема:
Катушка

А имеет собственную индуктивность 3 мГн, а ток через нее изменяется от 0,2 А. до 1,5 А за время 0,2 с. Найти величину средней наведенной ЭДС в катушке за это время.

Решение:

  • Рассуждение:
    ЭДС самоиндукции равна ЭДС = -L * ∆I / ∆t.
  • Детали расчета:
    L = 3 мГн, ∆I / ∆t = (1.5 А — 0,2 А) / 0,2 с = 6,5 А / с.
    э. Д. произвел это.
Проблема:

Круглая катушка с 25 витками проволоки имеет диаметр 1 м. Он размещен с его ось вдоль направления магнитного поля Земли (величина 50 мкТл), а затем за 0,2 с он переворачивается на 180 на . Какая средняя ЭДС сгенерировано

Решение:

  • Рассуждение:
    Φ B = B A — поток B через область A. Первоначально B и A выровнены, наконец, они анти-выровнены. Точка товар меняет знак.
  • Детали расчета:
    ЭДС = -∆Φ B / ∆t. Φ B (начальная) = NAB = 25 * π * (0,5 м) 2 50 * 10 -6 T = 9,82 * 10 -4 Tm 2 .
    Φ B (окончательный) = -Φ B (начальный), поскольку катушка перевернута.
    | ∆Φ B | = 2Φ B (начальное).
    | ∆Φ B / ∆t | знак равно 2 * (9.82 * 10 -4 Tm 2 ) / (0,2 с) = 9,82 * 10 -3 В.
Проблема:

Катушка с 500 витками радиусом 0,5 м поворачивается на четверть оборота за 4,17. мс, изначально имеющая плоскость, перпендикулярную однородному магнитному полю. Найдите напряженность магнитного поля, необходимую для индукции средней ЭДС 10 000 В.

Решение:

  • Рассуждение:
    ЭДС = -∆Φ B / ∆t. Φ B = NABcosθ изменяется с NAB на 0 в 4.17 мс, так как θ изменяется от 0 до 90 o через 4,17 мс.
  • Детали расчета:
    | ∆Φ B | = NAB = 500 * π * (0,5 м) 2 * B = (393 м 2 ) * Б.
    Хотим
    | emf | = | ∆Φ B / ∆t | = (393 м 2 ) / (4,17 * 10 -3 с) * B = (94174 м 2 / с) * B = 10000 В.
    B = 0,1 Вс / м 2 = 0,1 Т.

Если вы пропускаете регулярные лекции, обратите внимание на эту видеолекцию.

Лекция 16: Электромагнитная индукция

AP Physics — Блок 4

До сих пор мы видели электрические поля, создаваемые электрическими зарядами:

… и магнитные поля, создаваемые движущимися зарядами:

Вот вопрос: если токи создают магнитные поля, могут ли магнитные поля создавать токи?

Приступим!

  1. Закон индукции Фарадея
  2. Приложения
  3. Движение ЭДС
  4. Вихревые токи

Английский ученый и экспериментатор Майкл Фарадей в 1831 году разработал эксперимент, чтобы продемонстрировать, что изменяющееся магнитное поле создает поток заряда (ток).Независимо, американский ученый Джозеф Генри обнаружил тот же эффект в 1832 году.

Экспериментальная установка, показанная здесь, демонстрирует принцип электромагнитной индукции :

  1. В левой части схемы, когда переключатель замкнут, батарея вызывает прохождение тока в цепи. Катушка с проволокой, намотанная вокруг левой стороны тора, сначала не имела в ней тока и, следовательно, не имела магнитного поля.После включения переключателя в катушке, намотанной вокруг тора, появляется магнитное поле.
  2. Изменение магнитного поля в торе приводит к изменению магнитного потока в торе.
  3. В правой части схемы катушка с проволокой, намотанная вокруг тора, испытывает изменение магнитного потока, которое создает ЭДС в цепи справа, как описано в Законе электромагнитной индукции Фарадея.

Обратите внимание, что ЭДС сохраняется только до тех пор, пока магнитный поток продолжает изменяться.

ЭДС ε, индуцированная в цепи, прямо пропорциональна скорости изменения магнитного потока через цепь во времени.

Если мы хотим вызвать ток, нам нужно изменить магнитный поток, что мы можем сделать, изменив B , A или θ . Для некоторых проводящих контуров, таких как катушка с проволокой, N представляет количество катушек в контуре, что приводит к увеличению полезной площади контура.

Катушка намотана 200 витками проволоки по периметру квадратной рамки со сторонами 18 см. Общее сопротивление катушки 2,0 Ом. Включается однородное магнитное поле перпендикулярно плоскости катушки.

  1. Если поле изменяется линейно от 0 до 0,50 Вт / м2 за время 0,80 с, найдите величину наведенной ЭДС в катушке при изменении поля.
  2. Найдите величину тока, индуцируемого в катушке при изменении поля.

Решение

  1. Площадь = 0,0324 м 2 , ЭДС = 4,1 В
  2. 2,05 А

13.1.1. Закон Ленца

Закон Фарадея позволяет нам вычислить величину наведенной ЭДС (и, следовательно, тока), но для определения направления тока мы используем закон Ленца.

Закон Ленца гласит, что полярность наведенной ЭДС такова, что она создает ток, который будет создавать магнитный поток, чтобы противодействовал изменению магнитного потока через петлю.

На схеме проводящего кольца здесь, что произойдет, если магнитное поле, направленное вправо, начнет уменьшаться до ?

  1. Через проводник возникает магнитное поле, которое создает магнитный поток через область петли.
  2. По мере уменьшения поля уменьшается и поток.
  3. При изменении потока возникает ЭДС с током, но в каком направлении?
  4. Закон Ленца предсказывает индуцированный ток, который имеет собственное магнитное поле, противодействующее изменению потока.Что это обозначает?
  5. Если поток равен уменьшающимся для магнитного поля справа, индуцированный ток будет противодействовать этому уменьшающемуся потоку. Это произойдет, если индуцированный ток имеет магнитное поле в том же направлении, что и B .
  6. Направление тока, магнитное поле которого совпадает с направлением B , будет (согласно RHR) вниз спереди и вверх сзади. Следовательно, это направление тока, вызванного наведенной ЭДС.

На каждой из следующих диаграмм определите направление тока для ЭДС, индуцированной в проводящем контуре.

  1. Проводящая петля (зеленая) движется вправо, магнитное поле вправо
  2. Проводящая петля (зеленая) движется влево, магнитное поле вправо
  3. Проводящая петля (зеленая) движется влево, магнитное поле влево
  4. Вращающаяся проводящая петля, площадь петли в поле B уменьшается
  5. Вращающаяся проводящая петля, площадь петли в поле B увеличивается

Решения

  1. Вниз спереди, вверх сзади
  2. Вверху спереди, внизу сзади
  3. Вниз спереди, вверх сзади
  4. Вниз спереди, вверх сзади
  5. Вверху спереди, внизу сзади

Изучите следующую диаграмму.Найдите направление индуцированного тока в контуре:

  1. в момент включения переключателя
  2. после включения переключателя в течение нескольких секунд
  3. , когда переключатель был снова открыт

Решение

  1. Когда переключатель замкнут, ток начинает течь по петле, создавая увеличивающееся магнитное поле слева. Чтобы противодействовать этому изменению магнитного потока, ток в контуре индуцируется в направлении «вниз впереди, вверх сзади».
  2. Когда переключатель был замкнут в течение нескольких секунд, в контуре больше не течет ток, потому что больше нет изменения магнитного потока.
  3. Когда выключатель снова разомкнут, магнитное поле слева начинает уменьшаться. Петлевой ток противодействует этому уменьшению, имея направление, совпадающее с направлением магнитного поля, то есть «вверх спереди, вниз сзади».

13.1.2. Индуцированная ЭДС и электрическое поле

Мы определили, что изменяющийся магнитный поток индуцирует ЭДС и результирующий ток в проводящей петле, поэтому…. в проводнике должно быть электрическое поле. Мы не можем проанализировать это электрическое поле с помощью нашего электростатического анализа, потому что в тех ситуациях электрическое поле E было консервативным. Здесь, когда заряды движутся по кругу, индуцированное электрическое поле E неконсервативно .

Изменяющийся магнитный поток через проводник создает ЭДС, которая связана с изменяющимся во времени неконсервативным электрическим полем E .

Для проводящей петли радиусом r , подверженной изменяющемуся во времени магнитному полю B , ориентированному перпендикулярно плоскости петли, определяют величину изменяющегося электрического поля E в проводнике.

Решение

Электромагнитная индукция — это не просто тема эзотерической физики — она ​​используется в десятках приложений, от электрической зубной щетки до зарядного устройства для телефона, от розеток в стенах вашего дома до электрогитар.

Вы бы даже не сидели в хорошо освещенной комнате и читали это на компьютере, если бы Закон индукции Фарадея не давал вам возможности зажигать свет и управлять компьютером.

Вот несколько способов, которыми закон работает в некоторых окружающих нас устройствах.

13.2.1. Прерыватель замыкания на землю

Прерывание от замыкания на землю (GFI) — это стратегия отключения тока в цепи в аварийной ситуации.

  1. Обычно нет сетевого тока, заключенного в серый контур, поэтому нет магнитного поля вокруг проводов, а зеленая катушка, намотанная вокруг серого контура, не имеет ЭДС или тока.
  2. Если есть замыкание на землю (ток проходит через катушку в человека, например, но не возвращается по цепи), закон Ампера приводит к появлению магнитного поля в серой петле, увеличению магнитного потока и возникновению ЭДС в цепи. зеленая катушка, которая активирует автоматический выключатель, разрывая цепь.

Обычно GFI отключает питание цепи в течение 25-40 миллисекунд, предотвращая попадание значительного количества тока в тело человека, где это может помешать фибрилляционной активности сердца.

13.2.2. Звукосниматели для электрогитары

У акустической или классической гитары есть вибрирующие струны, которые вызывают резонанс полого корпуса гитары, который физически перемещает молекулы воздуха в области, воспринимаемой ушами как звук.

Некоторые акустические / классические гитары имеют небольшой микрофон, который улавливает звуковые волны, преобразует эти колебания в электрические сигналы и подает их на усилитель, который перемещает большой динамик с той же скоростью, что и гитарная струна, увеличивая громкость звука. без изменения его частоты.

Однако у электрогитары есть «звукосниматели», которые преобразуют звук в электрические сигналы для передачи на усилитель.

  1. Небольшой магнит в звукоснимателе находится под каждой металлической струной, намагничивая часть струны прямо над ней.
  2. Когда намагниченная струна начинает вибрировать, близлежащая катушка испытывает изменение магнитного потока с той же частотой, что и струна. Этот изменяющийся поток создает ЭДС / ток, который передается на усилитель.

Связанная с идеей ЭДС, создаваемая изменяющимся магнитным потоком, есть ЭДС, создаваемая движением проводника через магнитное поле, или ЭДС движения .

Когда мы перемещаем проводник через магнитное поле, возникает ЭДС движения . Рассмотрим проводящий стержень, движущийся в магнитном поле, как показано здесь. Что происходит с зарядами в проводнике при перемещении стержня вправо?

«Свободные заряды» в проводнике имеют скорость в результате движения через магнитное поле, поэтому они имеют скорость v и, таким образом, испытывают магнитную силу:

По правилу правой руки это заставляет положительные заряды выталкиваться вверх в стержне, а отрицательные — вниз.Это разделение зарядов приводит к возникновению электрического поля в стержне, вдали от положительных зарядов (направленного вниз), и это поле прикладывает электрическую силу для противодействия магнитной силе.

Заряды будут продолжать мигрировать вверх из-за магнитной силы, пока магнитная сила вверх и электрическая сила вниз не придет в динамическое равновесие.

На данный момент существует разница электрических потенциалов между верхним и нижним концами стержня, которую мы можем вычислить:

Это интересный результат, для понимания которого не требуется никаких знаний о магнитном потоке.Но давайте посмотрим, что произойдет с тем же примером, если мы установим проводящую полосу как движущийся компонент цепи.

Этот проводник, движущийся вправо со скоростью v , скользит по токопроводящим шинам, которые соединены резистором, образуя токопроводящую петлю. Согласно закону индукции Фарадея, изменение потока через этот контур вызовет ЭДС. Определите ЭДС в цепи как функцию заданных величин.

Решение

Обратите внимание, что этот результат тот же, к которому мы пришли при использовании двигательной ЭДС в предыдущем примере.

В каком направлении течет ток в этой петле?

Решение

  1. Определить направление магнитного поля: «на страницу»
  2. Определите, увеличивается или уменьшается поток в области: «увеличивается» по мере увеличения площади
  3. Используйте RHR, указав большим пальцем в противоположном направлении увеличения — «за пределы страницы» — для определения направления тока: «по часовой стрелке» или «вверх» на полосе.

Когда мы перемещаем этот стержень со скоростью v поперек магнитного поля, ЭДС заставляет ток проходить через стержень … и проводник с током в магнитном поле испытывает магнитную силу!

Давайте попробуем подсчитать, сколько силы, сколько энергии требуется, чтобы все это произошло, и сколько мощности рассеивается схемой.

Когда мы перемещаем стержень с током через магнитное поле, на него действует магнитная сила.Если мы перемещаем стержень с постоянной скоростью v по проводящим рельсам с незначительным трением, вычислим

  1. магнитная сила, действующая на стержень
  2. сила, которую мы используем для перемещения стержня
  3. мощность, рассеиваемая цепью

Решение

Мы уже определили, что наведенная в цепи ЭДС составляет Blv . Имея это в виду:

  1. Магнитная сила, действующая на стержень, равна
  2. сила, которую мы используем, чтобы переместить штангу
  3. мощность, рассеиваемая схемой, равна

Обратите внимание, что мощность, приложенная к эксперименту, и мощность, рассеиваемая схемой, одинаковы! Сохранение энергии!

Давайте посмотрим еще на одно интересное применение этой экспериментальной установки.

Стержне массой м и длиной l придают начальную скорость v 0 и отпускают так, что он скользит по двум параллельным рельсам без трения в присутствии магнитного поля B , как показано, с нагрузкой сопротивление R прилагается. Хотя нет силы трения, препятствующей движению, существует магнитная сила в направлении, противоположном v . Вычислите v как функцию времени t .

Решение

Магнитная сила, действующая на ток, индуцированный в стержне, действует в противоположном (отрицательном) направлении скорости:

Управляйте уравнениями, чтобы изолировать v с одной стороны и t с другой, а затем интегрировать по этим двум переменным:

или:

Если изменение магнитного потока вызывает ЭДС и токи в цепи, те же самые изменения магнитного потока могут вызвать вихревые токи (циркулирующие свободные заряды в массивном металле, движущиеся через магнитное поле).

В показанном здесь примере плоский проводящий прямоугольник прикреплен к рычагу маятника и качается вниз в поле силы тяжести. Когда свободные заряды в металле попадают в магнитное поле, они выталкиваются вверх в проводнике, в результате чего возникает круговой поток заряда в прямоугольнике. Ток в той части проводника, которая находится в магнитном поле, подвергается воздействию магнитной силы согласно F = Il × B , противоположной движению плеча маятника.

Что происходит, когда плоский проводящий прямоугольник начинает выходить из магнитного поля с противоположной стороны?

Поскольку вихревые токи создают тормозящие силы, их можно использовать в качестве тормозных систем для систем общественного транспорта. Если вихревые токи нежелательны, то объемный металл часто разделяют на тонкие слои, которые накладываются друг на друга.

Правило правой руки | PASCO

Правило правой руки в физике

Правило правой руки — это мнемоника руки, используемая в физике для определения направления осей или параметров, указывающих в трех измерениях.Правило правой руки, изобретенное в XIX веке британским физиком Джоном Амброузом Флемингом для применения в электромагнетизме часто используется для определения направления третьего параметра, когда известны два других (магнитное поле, ток, магнитная сила). Есть несколько вариантов правила правой руки, которые объясняются в этом разделе.

Когда проводник, такой как медный провод, движется через магнитное поле (B), в проводнике индуцируется электрический ток (I).Это явление известно как закон индукции Фарадея. Если проводник перемещается внутри магнитного поля, то существует соотношение между направлениями движения (скорости) проводника, магнитного поля и индуцированного тока. Мы можем использовать правило правой руки Флеминга исследовать закон индукции Фарадея, который представлен уравнением:

ЭДС = индуцированная ЭДС (V или J / C)
N = количество витков катушки
Δ𝚽 B = изменение магнитного потока (Тм2)
Δ t = изменение во времени (с)

Поскольку оси x, y и z перпендикулярны друг другу и образуют прямые углы, правило правой руки можно использовать для визуализации их выравнивание в трехмерном пространстве.Чтобы использовать правило правой руки, начните с создания L-образной формы с помощью большого пальца правой руки, указателя и середины. Палец. Затем переместите средний палец внутрь к ладони так, чтобы он был перпендикулярен указательным и большим пальцам. Твоя рука должно выглядеть примерно так:

На диаграмме выше большой палец совмещен с осью z, указательный палец — с осью x, а средний палец — с осью y.

Беспроводная интеллектуальная тележка

Один из лучших способов помочь учащимся обрести уверенность в использовании правила правой руки — это провести наглядную демонстрацию, которая поможет им распознать и исправить свои неправильные представления об ортогональных отношениях и системах координат.

Многие учителя используют вращающуюся линейку, чтобы показать, что объект, который кажется вращающимся «по часовой стрелке» с точки зрения одного ученика, также кажется вращающимся «против часовой стрелки», если смотреть с другой точки зрения. Использование динамической тележки для обучения правилу правой руки позволяет преподавателям продемонстрировать как проблему с помощью терминологии «по часовой стрелке», так и «против часовой стрелки», а также решение, которое обеспечивают правило правой руки и оси вращения. С беспроводной интеллектуальной тележкой преподаватели могут использовать 3-осевой гироскоп и фиксированную систему координат для создания увлекательных демонстраций вращательного движения.Ознакомьтесь с полной демонстрацией здесь.

Правило правой руки для магнетизма


Подвижные сборы

Заряженная частица — это частица с электрическим зарядом. Когда неподвижная заряженная частица существует в магнитном поле, она не испытать магнитную силу; однако, как только заряженная частица движется в магнитном поле, она испытывает наведенное магнитное поле. сила, которая смещает частицу с ее первоначального пути. Это явление, также известное как сила Лоренца, согласуется с правилом, что утверждает, что «магнитные поля не работают.”Уравнение, используемое для определения величины магнитной силы, действующей на заряженную частицу (q) перемещение магнитного поля (B) со скоростью v под углом θ составляет:

Если скорость заряженной частицы параллельна магнитному полю (или антипараллельна), то силы нет, потому что sin (θ) равен нулю. Когда это происходит, заряженная частица может сохранять прямолинейное движение даже в присутствии сильного магнитного поля.

Плоскость, образованная направлением магнитного поля и скоростью заряженной частицы, расположена под прямым углом к ​​силе.Поскольку сила возникает под прямым углом к ​​плоскости, образованной скоростью частицы и магнитным полем, мы можем использовать правило правой руки, чтобы определить их ориентацию.

Правило правой руки гласит: чтобы определить направление магнитной силы на положительный движущийся заряд, направьте большой палец правой руки в направление скорости (v), указательный палец в направлении магнитного поля (B) и средний палец будут указывать в направление результирующей магнитной силы (F).На отрицательные заряды будет действовать сила в противоположном направлении.

Магнитная сила, индуцированная током: ток в прямом проводе

Обычный ток состоит из движущихся зарядов, которые имеют положительный характер. Когда обычный ток проходит по проводящему проводу, на провод действует магнитное поле, которое его толкает. Мы можем использовать правило правой руки, чтобы определить направление силы, действующей на токоведущий провод. В этой модели ваши пальцы указывают в направлении магнитного поля, а большой палец — в направлении магнитного поля. обычный ток, протекающий через провод, и ваша ладонь указывает направление, в котором провод проталкивается (сила).

Магнитная сила, действующая на провод с током, определяется уравнением:

Когда длина провода и магнитное поле расположены под прямым углом друг к другу, уравнение принимает следующий вид:

F B = магнитная сила (Н)
I = ток (A)
L = длина провода (м)
B = магнитное поле (Тл)

Если рассматривать протекание тока как движение носителей положительного заряда (обычный ток) в приведенном выше image, мы замечаем, что обычный ток движется вверх по странице.Поскольку обычный ток состоит из положительных зарядов, то тот же провод с током также может быть описан как имеющий ток с отрицательным носители заряда движутся вниз по странице. Хотя эти токи движутся в противоположных направлениях, один наблюдается магнитная сила, действующая на провод. Следовательно, сила действует в том же направлении, независимо от того, рассмотрите поток положительных или отрицательных носителей заряда на изображении выше. Применение правила правой руки к направление обычного тока указывает направление магнитной силы, которое должно быть направлено вправо.Когда мы рассматриваем поток отрицательных носителей заряда на изображении выше, правило правой руки указывает на то, что направление силы, которую нужно оставить; однако отрицательный знак меняет результат на противоположный, указывая на то, что направление магнитной силы действительно указывает вправо.

Если мы рассмотрим поток зарядов в двух разных проводах, один с положительными зарядами, текущими вверх по странице, а другой с отрицательными зарядами, текущими вверх по странице, то направление магнитных сил не будет таким же, потому что мы рассматриваем две разные физические ситуации.В первом проводе поток положительных зарядов вверх по странице указывает на то, что по странице стекают отрицательные заряды. Правило правой руки говорит нам, что магнитный сила укажет в правильном направлении. По второму проводу вверх по странице текут отрицательные заряды, которые означает, что положительные заряды стекают по странице. В результате правило правой руки показывает, что магнитная сила указывает в левом направлении.

Токи, индуцированные магнитными полями

В то время как магнитное поле может быть индуцировано током, ток также может быть индуцирован магнитным полем.Мы можем использовать второе правило правой руки, иногда называемое правилом захвата правой руки, для определения направления магнитного поле, созданное током. Чтобы использовать правило захвата правой рукой, направьте большой палец правой руки в направлении течения. течь и скручивай пальцы. Направление ваших пальцев будет отражать направление искривления индуцированного магнитного поля.

Правило захвата правой рукой особенно полезно для решения проблем, связанных с токоведущим проводом или соленоидом. В обеих ситуациях правило захвата правой рукой применяется к двум приложениям закона оборота Ампера, который связывает интегрированное магнитное поле вокруг замкнутого контура к электрическому току, проходящему через плоскость замкнутого контура.

Направление вращения: соленоиды

Когда электрический ток проходит через соленоид, он создает магнитное поле. Чтобы использовать правило захвата правой рукой в проблема с соленоидом, укажите пальцами в направлении обычного тока и оберните пальцы, как будто они были вокруг соленоида. Ваш большой палец будет указывать в направлении силовых линий магнитного поля внутри соленоида. Примечание что силовые линии магнитного поля вне соленоида направлены в противоположном направлении. Они охватывают изнутри, чтобы снаружи соленоида.

Направление вращения: токоведущие провода

Когда электрический ток проходит по прямому проводу, он индуцирует магнитное поле. Чтобы применить правило захвата правой рукой, совместите большой палец с направлением обычного тока (от положительного к отрицательному), и ваши пальцы будут указывать направление магнитных линий потока.

Правило правой руки для крутящего момента


Проблемы с крутящим моментом часто являются самой сложной темой для студентов-первокурсников-физиков.К счастью, есть правило правой руки приложение для крутящего момента. Чтобы использовать правило правой руки в задачах с крутящим моментом, возьмите правую руку и наведите ее на направление вектора положения (r или d), затем поверните пальцы в направлении силы, и большой палец укажет в направлении крутящего момента.

Уравнение для расчета величины вектора крутящего момента для крутящего момента, создаваемого заданной силой:

Когда угол между вектором силы и плечом момента является прямым, синусоидальный член становится 1 и уравнение становится:

F = сила (Н)
𝜏 = крутящий момент (Нм)
r = расстояние от центра до линии действия (м)

Положительный и отрицательный крутящие моменты

Моменты, возникающие против часовой стрелки, являются положительными.В качестве альтернативы крутящие моменты, возникающие в по часовой стрелке — отрицательные моменты. Так что же произойдет, если ваша рука укажет на бумагу или из нее? Крутящие моменты, которые лицевой стороной наружу из бумаги следует анализировать положительный крутящий момент, в то время как крутящий момент, направленный внутрь, следует анализировать. как отрицательные моменты.

Правило правой руки для перекрестного произведения


Перекрестное произведение или векторное произведение создается, когда упорядоченная операция выполняется над двумя векторами, a и b. В векторное произведение векторов a и b перпендикулярно как a, так и b и перпендикулярно плоскости, которая его содержит.С есть два возможных направления для перекрестного произведения, для определения направления следует использовать правило правой руки вектора кросс-произведения.

Например, векторное произведение векторов a и b можно представить с помощью уравнения:

(произносится как «крест б»)

Чтобы применить правило правой руки к перекрестным произведениям, выровняйте пальцы и большой палец под прямым углом. Затем укажите свой индекс пальцем в направлении вектора a и средним пальцем в направлении вектора b.Ваш большой палец правой руки укажет в направлении векторного произведения a x b (вектор c).

Правило правой руки по закону Ленца


Закон электромагнитной индукции Ленца — еще одна тема, которая часто кажется нелогичной, поскольку требует понимание того, как магнетизм и электрические поля взаимодействуют в различных ситуациях. Закон Ленца гласит, что направление тока, индуцируемого в замкнутом проводящем контуре изменяющимся магнитным полем (закон Фарадея), такова, что вторичное магнитное поле, создаваемое индуцированным током, противодействует начальному изменению магнитного поля, которое произвело Это.Так что это значит? Давайте разберемся с этим.

Когда магнитный поток через проводник с замкнутым контуром изменяется, он индуцирует ток внутри контура. Индуцированная ток создает вторичное магнитное поле, которое противодействует первоначальному изменению потока, которое инициировало индуцированный ток. Сила магнитного поля, проходящего через катушку из проволоки, определяет магнитный поток. Магнитный поток зависит от сила поля, площадь катушки и относительная ориентация между полем и катушкой, как показано в следующем уравнении.


𝚽 B = магнитный поток (Tm 2 )
B = магнитное поле (Тл)
Θ = угол между полем и нормалью (град.)
A = площадь контура (м 2 )

Чтобы понять, как закон Ленца повлияет на эту систему, нам нужно сначала определить, является ли начальное магнитное поле увеличение или уменьшение силы. Когда северный магнитный полюс приближается к петле, это вызывает существующее магнитное поле. поле для увеличения.Поскольку магнитное поле увеличивается, индуцированный ток и результирующее индуцированное магнитное поле будут противодействовать исходному магнитному полю, уменьшая его. Это означает, что первичное и вторичное магнитные поля будут возникать в противоположные направления. Когда существующее магнитное поле уменьшается, индуцированный ток и результирующее индуцированное магнитное поле поле будет противодействовать исходному, уменьшая магнитное поле, усиливая его. Таким образом, индуцированное магнитное поле будет иметь в том же направлении, что и исходное магнитное поле.

Чтобы применить правило правой руки к закону Ленца, сначала определите, увеличивается ли магнитное поле, проходящее через петлю, или уменьшается. Напомним, что магниты создают силовые линии магнитного поля, которые движутся от северного магнитного полюса в направлении магнитный южный полюс. Если магнитное поле увеличивается, то направление вектора индуцированного магнитного поля будет в обратном направлении. Если магнитное поле в контуре уменьшается, то вектор индуцированного магнитного поля будет происходят в том же направлении, чтобы заменить уменьшение исходного поля.Затем выровняйте большой палец в направлении индуцированное магнитное поле и скрученные пальцы. Ваши пальцы будут указывать в направлении индуцированного тока.

Калькулятор закона Фарадея

Этот калькулятор закона Фарадея поможет вам найти электродвижущую силу, индуцированную в замкнутой цепи.

Что такое электромагнитная индукция?

Если вы прикрепите металлический провод к батарее, вы создадите ток — электроны будут двигаться по проводу. Тогда, если вы поместите этот провод в магнитное поле, дополнительная электрическая сила будет индуцирована движением электронов в этом поле.

Этот эффект также работает в обратном направлении — когда неподвижные электроны помещаются в переменное магнитное поле, индуцируется электродвижущая сила (ЭДС), и начинает течь ток. Это явление известно как электромагнитная индукция.

Магнитное поле и поток

Магнитное поле имеет две основные характеристики. Первый — это величина поля B и измеряется в теслах (символ Т), или в ньютонах на метр на ампер. Второй — магнитный поток Φ — определяется как магнитное поле, проходящее через поверхность, и измеряется в веберах (символ Wb).

Величина и поток взаимозависимы — вы можете использовать приведенное ниже уравнение, чтобы легко переключаться между ними. A обозначает площадь поперечного сечения катушки, в которой индуцируется ЭДС.

Φ = B * A

Tesla и веберы связаны следующей формулой:

1 Вт / 1 м² = 1 T

Закон Ленца и формула закона Фарадея

Закон Фарадея гласит, что индуцированное в цепи напряжение равно скорости изменения — то есть изменения во времени — магнитного потока через контур:

ЭДС = dΦ / dt

Закон Ленца — это второй ключевой закон, описывающий электромагнитную индукцию.Он не описывает величину, а скорее направление тока, утверждая, что ток всегда будет противодействовать потоку, который его произвел. Он включен в закон Фарадея со знаком минус:

ЭДС = - dΦ / dt

Если вы попытаетесь вызвать электродвижущую силу в катушке с несколькими витками, вы также можете умножить это значение на количество витков, чтобы учесть количество витков Н :

ЭДС = - N * dΦ / dt

Это формула, используемая нашим калькулятором закона Фарадея.

Как рассчитать электродвижущую силу?

Обычно вы не сразу узнаете, каков магнитный поток в катушке. Но не волнуйтесь — наш калькулятор электромагнитной индукции тоже может его найти! Просто выполните следующие действия, чтобы рассчитать наведенное напряжение.

  1. Определитесь с площадью поперечного сечения и количеством витков в петле. Например, вы можете использовать круглую катушку сечением 30 см² и десятью витками.

  2. Узнайте, какова величина магнитного поля.Например, мы можем принять поле в 0,4 тесла.

  3. Рассчитайте изменение магнитного потока как произведение магнитного поля на площадь поперечного сечения:

dΦ = B * A

dΦ = 0,4 * 30 * 10⁻⁴ = 0,0012 Wb

  1. Теперь определите, за сколько времени магнитное поле изменится на 0,4 Тл. Мы можем предположить, что это заняло 8 секунд.

  2. Используйте формулу закона Фарадея для вычисления электродвижущей силы:

ЭДС = - N * dΦ / dt

ЭДС = - 10 * 0.

Добавить комментарий

Ваш адрес email не будет опубликован. Обязательные поля помечены *